Vous êtes sur la page 1sur 95

MATHEMATICS

PROBLEM SOLVING ASSESSMENT

QUANTITATIVE APTITUDE

for

CLASS – IX and X
2014 – 15

IMPORTANT CONCEPTS TOPIC WISE ALONG


WITH SOLVED EXAMPLES

Prepared by

M. S. KUMARSWAMY, TGT(MATHS)
M. Sc. Gold Medallist (Elect.), B. Ed.

Kendriya Vidyalaya donimalai

Prepared by: M. S. KumarSwamy, TGT(Maths) Page - A -


PREFACE
It gives me great pleasure in presenting the Quantitative Aptitude Study material
for Problem Based Assessment (PSA) for Class IX and X Mathematics for II-term. It is in
strictly according to the latest guidelines issued by CBSE. The Central Board of
Secondary Education (CBSE) is planning to initiate a Problem Solving Assessment
(PSA) test for students of classes IX & XI. To be introduced from the second term of the
2012-2013 academic session, the tests aim to assess the students in the areas of
quantitative reasoning, qualitative reasoning and language conventions.

I avail this opportunity to convey my sincere thanks to respected sir, Shri S. Vijay
Kumar, Joint Commissioner(Admn), KVS HQ New Delhi, respected sir Shri Isampal,
Deputy Commissioner, KVS RO Bangalore, respected sir Shri P. V. Sairanga Rao, Deputy
Commissioner, KVS RO Varanasi, respected sir Shri P. Deva Kumar, Deputy Commissioner,
KVS RO Ahmedabad, respected sir Shri. K. L. Nagaraju, Assistant Commissioner, KVS RO
Bangalore, respected sir Shri.Gangadharaiah, Assistant Commissioner, KVS RO Bangalore,
respected sir Shri Sirimala Sambanna, Assistant Commissioner, KVS RO Hyderabad and
respected Shri M.K. Kulshreshtha, Assistant Commissioner, KVS RO Chandigarh for their
blessings, motivation and encouragement in bringing out this notes in such an excellent form.

I also extend my special thanks to respected madam Smt. Nirmala Kumari M.,
Principal, KV Donimalai and respected Shri. M. Vishwanatham, Principal, KV Raichur
for their kind suggestions and motivation while preparing this notes.

I would like to place on record my thanks to respected sir Shri. P. K. Chandran,


Principal, presently working in KV Bambolim. I have started my career in KVS under his
guidance, suggestions and motivation.

Inspite of my best efforts to make this notes error free, some errors might have
gone unnoticed. I shall be grateful to the students and teacher if the same are brought to
my notice. You may send your valuable suggestions, feedback or queries through email
to kumarsir34@gmail.com that would be verified by me and the corrections would be
incorporated in the next year Question Bank.

M. S. KUMARSWAMY

Prepared by: M. S. KumarSwamy, TGT(Maths) Page - B -


DEDICATED
TO
MY FATHER

LATE SHRI. M. S. MALLAYYA

Prepared by: M. S. KumarSwamy, TGT(Maths) Page - C -


NUMBERS

IMPORTANT FACTS AND FORMULAE

I. NUMERAL : In Hindu Arabic system, we use ten symbols 0, 1, 2, 3, 4, 5, 6,


7, 8, 9 called digits to represent any number.
A group of digits, denoting a number is called a numeral.
We represent a number, say 689745132 as shown below :

Ten Crores Ten Lacs Lacs Ten Thousands Hundreds Tens Units
7 5
Crores (10 ) (Millions) (10 ) Thousands (103 ) (102 ) (101) (100)
(108 ) (106) (104 )
6 8 9 7 4 5 1 3 2
We read it as : 'Sixty-eight crores, ninety-seven lacs, forty-five thousand, one
hundred and thirty-two'.

II PLACE VALUE OR LOCAL VALUE OF A DIGIT IN A NUMERAL :


In the above numeral :
Place value of 2 is (2 x 1) = 2; Place value of 3 is (3 x 10) = 30;
Place value of 1 is (1 x 100) = 100 and so on.
Place value of 6 is 6 x 108 = 600000000

III.FACE VALUE : The face value of a digit in a numeral is the value of the
digit itself at whatever place it may be. In the above numeral, the face value of 2
is 2; the face value of 3 is 3 and so on.

IV PRIME NUMBERS : A number greater than 1 is called a prime number, if it has


exactly two factors, namely 1 and the number itself.
Prime numbers upto 100 are : 2, 3, 5, 7, 11, 13, 17, 19, 23, 29, 31, 37, 41, 43,
47, 53, 59, 61, 67, 71, 73, 79, 83, 89, 97.
Prime numbers Greater than 100 : Let p be a given number greater than 100. To find
out whether it is prime or not, we use the following method :
Find a whole number nearly greater than the square root of p. Let k > p . Test
whether p is divisible by any prime number less than k. If yes, then p is not prime.
Otherwise, p is prime.
e.g,,We have to find whether 191 is a prime number or not. Now, 14 > 191 .
Prime numbers less than 14 are 2, 3, 5, 7, 11, 13.
191 is not divisible by any of them. So, 191 is a prime number.

V.TESTS OF DIVISIBILITY
1. Divisibility By 2 : A number is divisible by 2, if its unit's digit is any of 0, 2, 4, 6,
8. Ex. 84932 is divisible by 2, while 65935 is not.

2. Divisibility By 3 : A number is divisible by 3, if the sum of its digits is divisible


by 3. Ex.592482 is divisible by 3, since sum of its digits = (5 + 9 + 2 + 4 + 8 + 2)
= 30, which is divisible by 3. But, 864329 is not divisible by 3, since sum of its
digits =(8 + 6 + 4 + 3 + 2 + 9) = 32, which is not divisible by 3.

3. Divisibility By 4 : A number is divisible by 4, if the number formed by the last


two digits is divisible by 4. Ex. 892648 is divisible by 4, since the number formed
Prepared by: M. S. KumarSwamy, TGT(Maths) Page - 1 -
by the last two digits is 48, which is divisible by 4. But, 749282 is not divisible
by 4, since the number formed by the last tv/o digits is 82, which is not divisible
by 4.
4. Divisibility By 5 : A number is divisible by 5, if its unit's digit is either 0 or 5.
Thus, 20820 and 50345 are divisible by 5, while 30934 and 40946 are not.
5. Divisibility By 6 : A number is divisible by 6, if it is divisible by both 2 and 3.
Ex. The number 35256 is clearly divisible by 2. Sum of its digits = (3 + 5 + 2 + 5
+ 6) = 21, which is divisible by 3. Thus, 35256 is divisible by 2 as well as 3.
Hence, 35256 is divisible by 6.
6. Divisibility By 8 : A number is divisible by 8, if the number formed by the last
three digits of the given number is divisible by 8. Ex. 953360 is divisible by 8,
since the number formed by last three digits is 360, which is divisible by 8. But,
529418 is not divisible by 8, since the number formed by last three digits is 418,
which is not divisible by 8.

7. Divisibility By 9 : A number is divisible by 9, if the sum of its digits is divisible


by 9. Ex. 60732 is divisible by 9, since sum of digits * (6 + 0 + 7 + 3 + 2) = 18,
which is divisible by 9. But, 68956 is not divisible by 9, since sum of digits = (6 +
8 + 9 + 5 + 6) = 34, which is not divisible by 9.
8. Divisibility By 10 : A number is divisible by 10, if it ends with 0. Ex. 96410,
10480 are divisible by 10, while 96375 is not.

9. Divisibility By 11 : A number is divisible by 11, if the difference of the sum of its


digits at odd places and the sum of its digits at even places, is either 0 or a number
divisible by 11. Ex. The number 4832718 is divisible by 11, since : (sum of digits
at odd places) - (sum of digits at even places) (8 + 7 + 3 + 4) - (1 + 2 + 8) = 11,
which is divisible by 11.

10. Divisibility By 12 ; A number is divisible by 12, if it is divisible by both 4 and 3.


Ex. Consider the number 34632.
(i) The number formed by last two digits is 32, which is divisible by 4,
(ii) Sum of digits = (3 + 4 + 6 + 3 + 2) = 18, which is divisible by 3. Thus, 34632
is divisible by 4 as well as 3. Hence, 34632 is divisible by 12.

11. Divisibility By 14 : A number is divisible by 14, if it is divisible by 2 as well as 7.

12. Divisibility By 15 : A number is divisible by 15, if it is divisible by both 3 and 5.

13. Divisibility By 16 : A number is divisible by 16, if the number formed by the


last4 digits is divisible by 16. Ex.7957536 is divisible by 16, since the number
formed by the last four digits is 7536, which is divisible by 16.

14. Divisibility By 24 : A given number is divisible by 24, if it is divisible by both3


and 8.
15. Divisibility By 40 : A given number is divisible by 40, if it is divisible by both
5 and 8.
16. Divisibility By 80 : A given number is divisible by 80, if it is divisible by both 5
and 16.
Note : If a number is divisible by p as well as q, where p and q are co-primes, then

Prepared by: M. S. KumarSwamy, TGT(Maths) Page - 2 -


the given number is divisible by pq. If p arid q are not co-primes, then the given
number need not be divisible by pq, even when it is divisible by both p and q. Ex.
36 is divisible by both 4 and 6, but it is not divisible by (4x6) = 24, since
4 and 6 are not co-primes.

VI. BASIC FORMULAE


1. (a + b)2 = a2 + b2 + 2ab 2. (a - b)2 = a2 + b2 - 2ab
2 2
3. (a + b) - (a - b) = 4ab 4. (a + b)2 + (a - b)2 = 2 (a2 + b2)
5. (a2 - b2) = (a + b) (a - b)
6. (a + b + c)2 = a2 + b2 + c2 + 2 (ab + bc + ca)
7. (a3 + b3) = (a +b) (a2 - ab + b2) 8. (a3 - b3) = (a - b) (a2 + ab + b2)
9. (a + b + c -3abc) = (a + b + c) (a + b2 + c2 - ab - bc - ca)
3 3 3 2

10. If a + b + c = 0, then a3 + b3 + c3 = 3abc.

VII. DIVISION ALGORITHM OR EUCLIDEAN ALGORITHM


If we divide a given number by another number, then :
Dividend = (Divisor x Quotient) + Remainder

VIII. {i) (xn - an ) is divisible by (x - a) for all values of n.


(ii) (xn - an) is divisible by (x + a) for all even values of n.
(iii) (xn + an) is divisible by (x + a) for all odd values of n.

IX. PROGRESSION
A succession of numbers formed and arranged in a definite order according to certain
definite rule, is called a progression.

Arithmetic Progression (A.P.) : If each term of a progression differs from its


preceding term by a constant, then such a progression is called an arithmetical
progression. This constant difference is called the common difference of the A.P.
An A.P. with first term a and common difference d is given by a, (a + d), (a + 2d),(a +
3d),.....
The nth term of this A.P. is given by Tn =a + (n – 1) d.
The sum of n terms of this A.P.
n n n
S n   2a  (n  1)d   (a  an )  (a  l )
2 2 2

SOME IMPORTANT RULES:

n(n  1)
Rule 1: Sum of first n natural numbers :
2
Rule 2: Sum of first n odd numbers : n x n = n2.
Rule 3: Sum of first n even numbers = n(n+1)
n(n  1)(2n  1)
Rule 4: Sum of squares of first n natural numbers :
6
2
 n(n  1) 
Rule 5: Sum of cubes of first n natural numbers :  
 2 
Rule 6: If n is the number of numbers and n is even then n/2 numbers will be
even and n/2 numbers will be odd among first n natural numbers.
Rule 7: If n is odd , then there are (n+1)/2 odd numbers and (n-1)/2 even
numbers
Rule 8: The difference between the squares of two consecutive numbers is always
Prepared by: M. S. KumarSwamy, TGT(Maths) Page - 3 -
an odd number. Ex. 92 – 82 = 17
Rule 9: The difference between the squares of two consecutive numbers is the
sum of the two consecutive numbers

Ex. Find out the number of all even numbers from 1 to 300 ?
Solution : Since 300 is an even number so total number of even numbers wil be (n/20)
= (300/2) = 150 even numbers

Ex. What is the sum of all the even numbers from 1 to 381
Solution : Even numbers will be = (381-1)/2= 190
Sum of even numbers = n * (n+1) = 190 (190+1) = 36,290

Ex. Find out of sum of all the odd numbers from 50 to 200
Solution : Required Sum = Sum of all odd numbers from 1 to 200 - Sum of all odd
numbers from 1 to 50 : 1002 – 252 = 9375
Rule 10 : Dividend = (Divisor * Quotient) + Remainder

Ex. What least number must be added to 7963 to make it exactly divisible by 65 ?
Solution :On dividing 7963 by 65 we get 33 as remainder, So the number to be added
will be 65 - 33 = 32

Ex. What least number must be subtracted from 7963 to make it exactely
divisible by 65 ?
Solution : On dividing 7963 by 65 we get 33 as remainder, So the number to be
subtracted will be 33

Ex. Find the least number of five digits which is exactly divisible by 73 ?
Solution :Least number of five digits will be 10000, on dividing 10000 by 73 we get
72 as remainder, so the number will be = 10000 + 72 = 10072

Rule : for finding least number add the remainder to the least number

Ex. find the greatest number of five digits which is exactly divisible by 147 ?
Solution : The greatest number of five digit will be 99999, on dividing it by 147 we
get 39 as remainder, so the required number will be 99999 - 39 = 99960

Rule : For finding greatest number substract the remainder to the greatest
number

H.C.F. AND L.C.M. OF NUMBERS


IMPORTANT FACTS AND FORMULAE

I. Factors and Multiples : If a number a divides another number b exactly, we say


that a is a factor of b. In this case, b is called a multiple of a.
II. Highest Common Factor (H.C.F.) or Greatest Common Measure (G.C.M.) or
Greatest Common Divisor (G.C.D.): The H.C.F. of two or more than two numbers
is the greatest number that divides each of them exactly.
There are two methods of finding the H.C.F. of a given set of numbers :
1. Factorization Method : Express each one of the given numbers as the
product of prime factors.The product of least powers of common prime factors
gives H.C.F.
Prepared by: M. S. KumarSwamy, TGT(Maths) Page - 4 -
2. Division Method: Suppose we have to find the H.C.F. of two given
numbers. Divide the larger number by the smaller one. Now, divide the divisor
by the remainder. Repeat the process of dividing the preceding number by the
remainder last obtained till zero is obtained as remainder. The last divisor is the
required H.C.F.
Finding the H.C.F. of more than two numbers : Suppose we have to find the
H.C.F. of three numbers. Then, H.C.F. of [(H.C.F. of any two) and (the third
number)] gives the H.C.F. of three given numbers.
Similarly, the H.C.F. of more than three numbers may be obtained.
III. Least Common Multiple (L.C.M.) : The least number which is exactly divisible
by each one of the given numbers is called their L.C.M.
1. Factorization Method of Finding L.C.M.: Resolve each one of the given
numbers into a product of prime factors. Then, L.C.M. is the product of
highest powers of all the factors,
2. Common Division Method {Short-cut Method) of Finding L.C.M.:
Arrange the given numbers in a row in any order. Divide by a number which
divides exactly at least two of the given numbers and carry forward the
numbers which are not divisible. Repeat the above process till no two of the
numbers are divisible by the same number except 1. The product of the
divisors and the undivided numbers is the required L.C.M. of the given
numbers,

IV. Product of two numbers =Product of their H.C.F. and L.C.M.

V. Co-primes: Two numbers are said to be co-primes if their H.C.F. is 1.

VI. H.C.F. and L.C.M. of Fractions:


1.H C F= H.C.F. of Numerators 2.L C M = L.C.M of Numerators__
L.C.M. of Denominators H.C.F. of Denominators
VII. H.C.F. and L.C.M. of Decimal Fractions: In given numbers, make the same
number of decimal places by annexing zeros in some numbers, if necessary.
Considering these numbers without decimal point, find H.C.F. or L.C.M. as the case
may be. Now, in the result, mark off as many decimal places as are there in each of
the given numbers.
VIII. Comparison of Fractions: Find the L.C.M. of the denominators of the given
fractions. Convert each of the fractions into an equivalent fraction with L.C.M. as the
denominator, by multiplying both the numerator and denominator by the same
number. The resultant fraction with the greatest numerator is the greatest.

MCQ QUESTIONS FOR PRACTICE

1. Find the least number which when divided by 6,15 and 18 leave remainder 5 in
each case
(a) 90 (b) 180 (c) 95 (d)185

2. Divisibility by 2,5,10 can be checked by


(a) sum of digits (b) last digit (c) last two digits (d) last three digits

3. Which is greatest 3-digit number exactly divisible by 8,10,12


(a) 120 (b) 360 (c) 960 (d) 980

Prepared by: M. S. KumarSwamy, TGT(Maths) Page - 5 -


4. 4= 2x2 , 15= 3x5, so H.C.F. of 4 and 15 is
(a) 0 (b) 1 (c) 2 (d) 3
5. Find the least number which when divided by 12, 16, 24 and 36 leaves a
remainder 7 in each case.
(a) 150 (b) 151 (c)144 (d) none of these

6. Renu purchases two bags of fertiliser of weights 75 kg and 69 kg. Find the
maximum value of weight which can measure the weight of the fertiliser exact
number of times.
(a) 150 (b) 138 (c)144 (d) none of these

7. Which of the following is divisible by 3?


(a)15287 (b) 15267 (c) 15286 (d) 152638

8. Which of the following is divisible by 9?


(a)15287 (b) 15267 (c) 15286 (d) 152638

9. If a number is divisible by 9, it must be divisible by __.


(a) 6 (b) 3 (c) 2 (d) 12

10. The HCF of smallest composite number and the smallest prime number is
(a) 0 (b) 1 (c) 2 (d) 3

11. Given that HCF(1152, 1664) = 128 the LCM(1152, 1664) is


(a) 14976 (b) 1664 (c) 1152 (d) none of these

12. The HCF of two numbers is 23 and their LCM is 1449. If one of the numbers is
161, then the other number is
(a) 23 (b) 207 (c) 1449 (d) none of these

13. Which one of the following rational number is a non-terminating decimal


expansion:
33 66 6 41
(a) (b) (c) (d)
50 180 15 1000

14. What is the H.C.F. of two consecutive even numbers


(a) 1 (b)2 (c) 4 (d) 8

15. What is the H.C.F. of two consecutive odd numbers


(a) 1 (b) 2 (c) 4 (d) 8
16. The missing number in the following factor tree is
(a) 2 (b) 6 (c) 3 (d) 9

17. If the HCF of 65 and 117 is expressible in the form 65m – 117, then the value of m
is
(a) 4 (b) 2 (c) 1 (d) 3

Prepared by: M. S. KumarSwamy, TGT(Maths) Page - 6 -


18. The largest number which divides 70 and 125, leaving remainders 5 and 8,
respectively, is
(a) 13 (b) 65 (c) 875 (d) 1750

19. If two positive integers a and b are written as a = x3y2 and b = xy3 ; x, y are prime
numbers, then HCF (a, b) is
(a) xy (b) xy2 (c) x3y3 (d) x2y2

20. If two positive integers p and q can be expressed as p = ab2 and q = a3b; a, b being
prime numbers, then LCM (p, q) is
(a) ab (b) a2b2 (c) a3b2 (d) a3b3

21. HCF of 256, 320 is


a) 32 b) 16 c) 64 d) 4

22. If X = 25 × 3, Y = 24 × 3 × 11, Z = 24 × 32 × 11 then the G.C.D of X, Y and Z is


a) 24 × 3 b) 25 × 3 c) 24 × 32 d) 25 × 32

23. Observe the process and find the numbers x, y, z

a) x = 1, y = 2, z = 14 b) x = 1, y = 1,z = 14
c) x = 2, y = 1, z = 14 d) x = 2, y = 2, z = 14

24. Find the largest number which can exactly divide 513, 783 and 1107
a) 27 b) 26 c) 28 d) 29

25. Three drums contain 72 litres, 108 litres and 144 litres of oil. What biggest
measure can measure all the different quantities exactly
a) 32 litres b) 38 litres c) 36 litres d) 34 litres

26. Find the smallest number exactly divisible by 12, 15, 20 and 27
a) 520 b) 560 c) 540 d) 580

27. LCM of 24, 54 and 72 is


a) 214 b) 218 c) 216 d) 224

28. If A = 37 × 48 × 53, B = 36 × 43 × 58 then LCM of A and B is


a) 37 × 48 × 53 b) 36 × 43 × 58 c) 37 × 48 × 58 d) None of these

29. Find the least number when divided by 6, 7, 8, 9 and 12 leaves the same
remainder 2 in each case
a) 508 b) 504 c) 506 d) 502

30. HCF of three numbers is 12. If they be in the ratio 2 : 3 : 4 then the numbers

Prepared by: M. S. KumarSwamy, TGT(Maths) Page - 7 -


a) 24, 36, 48 b) 12, 18, 24 c) 10, 20, 30 d) None of these

31. The sum of the two numbers is 216 and their HCF is 27. The numbers are
a) 54, 162 b) 27, 189 c) 108, 108 d) None of these

32. Three different containers contain different quantities of mixtures of milk and
water, whose measurements are 384kg, 432kg and 480kg. What is the biggest
measure must be there to measure all the different quantities exactly
a) 48kg b) 46kg c) 42kg d) 44kg

33. The least number of square tiles required to pave the ceiling of a room 15m 60cm
and 9m 10 cm
a) 86 b) 88 c) 84 d) 82

34. The largest number which divides 147, 252, 282 to leave the same remainder
a) 9 b) 15 c) 25 d) 35

35. The largest natural number which exactly divides the product of any consecutive
natural numbers is
a) 6 b) 12 c) 24 d) 120

36. The least multiple of 7, which leaves a remainder 4, when divided by 6, 9, 15 and
18 is
a) 94 b) 194 c) 364 d) 414

37. Six sirens commence tolling together and toll at intervals of 2, 4, 6, 8, 10 and 12
seconds respectively. In 40 minutes how many times, do they toll together.
a) 21 times b) 22 times c) 23 times d) 24 times

38. The least number which when divided by 16, 18 and 21 leaves the remainders 3,
5, 8 respectively
a) 996 b) 998 c) 995 d) 997

39. The HCF and LCM of two numbers are 44 and 264. If the first number is divided
by 2 the quotient is 44. The other number is
a) 134 b) 136 c) 130 d) 132

40. What least number must be subtracted from 1294 so that the remainder when
divided by 9, 11, 13 will leave in each case the same remainder 6?
a) 0 b) 1 c) 2 d) 3

41. The product of two numbers is 1200 and their HCF is 5. The LCM of the numbers
a) 220 b) 260 c) 240 d) 280

42. The HCF of two numbers is 12 and their difference is also 12. The numbers are
a) 66, 78 b) 70, 82 c) 94, 106 d) 96, 108

43. About the number of pairs which have 16 as their HCF and 126 as their LCM.,
We can definitely say that.
a) Only one pair exists b) Only two pairs exist.
c) Many such pairs exist d) No such pair exists

Prepared by: M. S. KumarSwamy, TGT(Maths) Page - 8 -


1 3 5 7 9
44. HCF of , , , , is
2 4 6 8 10
1 1 9 1
a) b) c) d)
2 10 120 120

2 4 5 10
45. What is the LCM of , , , ?
3 9 6 15
10 40 20 80
a) b) c) d)
3 3 3 3

DECIMAL FRACTIONS
IMPORTANT FACTS AND FORMULAE
I. Decimal Fractions : Fractions in which denominators are powers of 10 are
known as decimal fractions.
Thus ,1/10=1 tenth=.1;1/100=1 hundredth =.01;
99/100=99 hundreths=.99;7/1000=7 thousandths=.007,etc
II. Conversion of a Decimal Into Vulgar Fraction : Put 1 in the denominator under the
decimal point and annex with it as many zeros as is the number of digits after the
decimal point. Now, remove the decimal point and reduce the fraction to its lowest
terms.
25 1 2008 251 1.84 184 8 0.365 365
Thus, 0.25   ; 2.008   .   ; 
100 4 1000 125 2.99 299 13 0.584 584
III. 1. Annexing zeros to the extreme right of a decimal fraction does not change its
value. Thus, 0.8 = 0.80 = 0.800, etc.
2. If numerator and denominator of a fraction contain the same number of
decimal places, then we remove the decimal sign.
1.84 184 8 0.365 365
Thus,   ; 
2.99 299 13 0.584 584
IV. Operations on Decimal Fractions :
1. Addition and Subtraction of Decimal Fractions : The given numbers
are so placed under each other that the decimal points lie in one
column. The numbers so arranged can now be added or subtracted in
the usual way.
2. Multiplication of a Decimal Fraction By a Power of 10 : Shift the
decimal point to the right by as many places as is the power of 10.
Thus, 5.9632 x 100 = 596,32; 0.073 x 10000 = 0.0730 x 10000 = 730.
3. Multiplication of Decimal Fractions : Multiply the given numbers
considering them without the decimal point. Now, in the product, the
decimal point is marked off to obtain as many places of decimal as is
the sum of the number of decimal places in the given numbers.
Suppose we have to find the product (.2 x .02 x .002).
Now, 2x2x2 = 8. Sum of decimal places = (1 + 2 + 3) = 6.
.2 x .02 x .002 = .000008.
4. Dividing a Decimal Fraction By a Counting Number : Divide the
given number without considering the decimal point, by the given
counting number. Now, in the quotient, put the decimal point to give as
many places of decimal as there are in the dividend. Suppose we have
to find the quotient (0.0204 + 17). Now, 204 ^ 17 = 12. Dividend
contains 4 places of decimal. So, 0.0204 + 17 = 0.0012.
5. Dividing a Decimal Fraction By a Decimal Fraction : Multiply both
the dividend and the divisor by a suitable power of 10 to make divisor
a whole number. Now, proceed as above.
Prepared by: M. S. KumarSwamy, TGT(Maths) Page - 9 -
Thus, 0.00066/0.11 = (0.00066*100)/(0.11*100) = (0.066/11) = .006V
V. Comparison of Fractions : Suppose some fractions are to be arranged in
ascending or descending order of magnitude. Then, convert each one of the given
fractions in the decimal form, and arrange them accordingly.
Suppose, we have to arrange the fractions 3/5, 6/7 and 7/9 in descending order.
now, 3/5=0.6,6/7 = 0.857,7/9 = 0.777....
since 0.857>0.777...>0.6, so 6/7>7/9>3/5
VI. Recurring Decimal : If in a decimal fraction, a figure or a set of figures is
repeated continuously, then such a number is called a recurring decimal.
In a recurring decimal, if a single figure is repeated, then it is expressed by putting
a dot on it. If a set of figures is repeated, it is expressed by putting a bar on the set
Thus 1/3 = 0.3333….= 0.3; 22 /7 = 3.142857142857.....= 3.142857
Pure Recurring Decimal: A decimal fraction in which all the figures after the
decimal point are repeated, is called a pure recurring decimal.
Converting a Pure Recurring Decimal Into Vulgar Fraction : Write the
repeated figures only once in the numerator and take as many nines in the
denominator as is the number of repeating figures.
thus ,0.5 = 5/9; 0.53 = 53/59 ;0.067 = 67/999;etc...
Mixed Recurring Decimal: A decimal fraction in which some figures do not
repeat and some of them are repeated, is called a mixed recurring decimal.
e.g., 0.17333 = 0.173.
Converting a Mixed Recurring Decimal Into Vulgar Fraction : In the
numerator, take the difference between the number formed by all the digits after
decimal point (taking repeated digits only once) and that formed by the digits
which are not repeated, In the denominator, take the number formed by as many
nines as there are repeating digits followed by as many zeros as is the number of
non-repeating digits.
Thus 0.16 = (16-1) / 90 = 15/19 = 1/6;
0.2273  (2273  22) / 9900  2251/ 9900

MCQ QUESTIONS FOR PRACTICE

9
1. What is the decimal expansion of
1000
a) 0.9 b) 9000 c) 0.009 d) 0.09

4 1
2. 20+9+ + can be written in decimal as
10 100
a) 29.04 b) 29.40 c) 2940 d) 0.2940

7 6 4
3. The decimal form + + can be written as
10 100 1000
a) 76.40 b) 7.640 c) 0.764 d) 764.0

9
4. 700 +20 + 5 + can be written in decimal form as
100
a) 725.09 b) 725.9 c) 72.59 d) 7.259

9
5. 70 +2 + can be written in decimal form as
100
a) 72.9 b) 729 c) 72.09 d) 7.209

Prepared by: M. S. KumarSwamy, TGT(Maths) Page - 10 -


2
6. 30  6  can be written in decimal form as
10
a) 3062 b) 362 c) 36.2 d) none of these

8
7. 600  2  can be written in decimal form as
10
a) 6002.8 b) 602.8 c) 628 d) none of these

8
8. 60  2  can be written in decimal form as
100
a) 62.8 b) 62.008 c) 62.08 d) none of these

9. The sum of 0.007 + 8.5 +30.08 is


a) 38.587 b) 3.100 c) 18.508 d) 385.87

10. Lata spend Rs 9.50 for buying a pen and Rs 2.50 for one pencil .How much
money did she spend
a) Rs 3.450 b) Rs 7 c) Rs 9.750 d) Rs 12

11. Find the value of 9.756 – 6.28


a)16.036 b)9.128 c)3.476 d)34.76

12. Find the value of 35 – 2.54


a)32.46 b)1.46 c)3.246 d)37.54

13. Subtract Rs. 18.25 from Rs. 20.75


a)Rs. 25 b) Rs. 39 c) Rs. 2.50 d)Rs. 3.9

14. Raju bought a book for Rs. 35.65. He gave Rs. 50 to the shopkeeper. How much
money did he get back from the shopkeeper?
a)Rs. 36.15 b)Rs. 14.35 c)Rs. 80.65 d) Rs. 1.435

15. Akash bought vegetables weighing 10kg.Out of this 3kg 500g is onions, 2kg 75g
is tomatoes and the rest is potatoes. What is the weight of the potatoes?
a)9.500kg b) 1.425kg c)5.575kg d)4.425kg

16. The number 0.125 can be written as fractions in lowest terms


1 125 25 5
a) b) c) d)
8 1000 200 40
17. 1mm = _____
a)0.1cm b)0.01 cm c) 1.0 cm d)0.001cm

18. Which one of the following is not true


a)1.431 < 1.490 b)3.3 > 3.300 c)0.3 < 0.4 d) 3 > 0.8

19. The length of a young gram plant is 65mm. its length in cm will be
a)6.5cm b)0.65cm c)0.065cm d)6.05 cm.
20. The length of Ramesh’s notebook is 9 cm 5 mm. What will be its length in cm?
a)9.5cm b)0.95cm c)0.095cm d)9.05 cm.

21. Write 2.34 as fractions in lowest terms.


17 171 17
a) b) 2 c) 2 d) none of these
20 500 20

Prepared by: M. S. KumarSwamy, TGT(Maths) Page - 11 -


22. Write 0.342 as fractions in lowest terms.
171 171 17
a) 2 b) c) d) none of these
500 500 20
23. 725 Paisa in rupees can be written as
a) 72.5 b) 0.725 c) 7.25 d) 0.0725
24. 4.19 m in cm can be written as
a) 419cm b) 41.9cm c) 0.419cm d) 41.09cm
25. 8888m in Km can be written as
a) 88.88Km b) 888.8Km c) 8.888Km d) 8888Km

26. 22g in Kg can be written as


a) 2.2Kg b) 0.022Kg c) 2.002Kg d) 2.02Kg

3 4 6
27. 1000  500  60  9    is the expended form of
10 1000 10000
a) 1569.3406 b) 1569.346 c) 1569.3046 d)
1569.643

28. If 78.36 = 78.36xy then x and y ?


a) 1 each b) 0 each c) 6 each d) 3 each

29. If 15 + 2.094 + 3.62 = X then X is


a) 20.724 b) 20.704 c) 20.714 d) 20.734

30. If 3.7 + 4.96 + 13.578+ 12.1347 = Y then Y + 99 =


a) 134.3727 b) 135.3727 c) 133.3727 d) 132.3727

31. If X = 3.7 + 2.95, Y = 8.7 + 3.999, Z = 10 then X + Y + Z =


a) 28.349 b) 30.349 c) 29.349 d) 27.349

32. When a decimal is multiplied by some power of 10 then the decimal point is
shifted to the____by as many digits as there are zeros in the multiplier ?
a) left b) right c) top d) below

33. If 2.3 × x = 46, 6.9 × y = 138 then x × y × 0.138 =


a) 55.2 b) 56.2 c) 57.2 d) 54.2

34. Match the following:


a 14 – 2.9 p 13.907
b 25 – 12.09 q 11.1
c 36 – 22.009 r 13.991
d 46 – 32.093 s 12.91
a) a – p, b – r, c – q, d – s b) a – p, b – r, c – q, d – s
c) a – q, b – s, c – r, d – p d) a – p, b – r, c – q, d – s

35. If A = 201.4 – 132.68; B = 11.62 – 6.068 then A – B =


a) 63.178 b) 63.158 c) 63.168 d) 63.188

Prepared by: M. S. KumarSwamy, TGT(Maths) Page - 12 -


36. Match the following:
a 98.7034 × 10 p 987034
b 98.7034 × 1000 q 9870.34
c 98.7034 × 100 r 98703.4
d 98.7034 × 10000 s 987.034
a) a – s, b – r, c – q, d – p b) a – p, b – r, c – q, d – s
c) a – p, b – r, c – q, d – s d) a – p, b – r, c – q, d – s

37. What should be added to 78.3091 to get 93?


a) 14.6809 b) 14.6709 c) 14.6909 d) 14.7009

38. What should be subtracted from 82 to get 73.22?


a) 8.68 b) 8.98 c) 8.88 d) 8.78

39. What is the excess of 90 over 67.214?


a) 21.786 b) 22.786 c) 23.786 d) 24.786

40. If 6258 × 74 = 463092 then 6.258 × 7.4 =


a) 46.3092 b) 4.63092 c) 463.092 d) 4630.92

41. If 0.4325 × 15 = 6.4875 then 43.25 × 1.5 =


a) 64.875 b) 648.75 c) 6.4875 d) 0.64875

42. If 6.57 × 12 = 78.84 then 0.0657 × 0.12 =


a) 0.07884 b) 0.7884 c) 0.007884 d) 0.0007884

43. If 0.0760 × 0.89 = 0.06764 then 7.6 × 0.00089 =


a) 6.764 b) 0.06764 c) 0.6764 d) 0.006764

44. Match the following:


a 3.7 + 2.99 p 6.001
b 0.47 + 3.3 q 4.19
c 0.5 + 3.69 r 3.77
d 3.47 + 2.531 s 6.69
a) a – p, b – r, c – q, d – s b) a – s, b – p, c – q, d – r
c) a – s, b – r, c – q, d – p d) a – r, b – q, c – q, d – s

45. If 48.47 × 30 = X, 29.07 × 400 = Y then X + Y =


a) 13072.1 b) 13082.1 c) 13062.1 d) 13092.1

x y z x y z 6
46. If    0.0478 then    
100 1000 10000 10 100 1000 10000

a) 0.4786 b) 0.4768 c) 0.4687 d) 0.4687

Prepared by: M. S. KumarSwamy, TGT(Maths) Page - 13 -


7.6  3.1
47. 
1.9
a) 12.6 b) 12.8 c) 12.2 d) 12.4

0.04  0.16
48. 
0.24  0.36
1 2 2 1
a) b) c) d)
26 29 27 29

49.
 2.5  0.04

7.5  0.125
1 1 1 1
a) b) c) d)
4 6 5 10

4.5  0.25
50.  0.03  10.4 
0.225
a) 4.658 b) 4.778 c) 4.688 d) 4.798

SIMPLIFICATION

IMPORTANT FACTS AND FORMULAE


I. ’BODMAS’Rule: This rule depicts the correct sequence in which the operations
are to be executed,so as to find out the value of a given expression.

Here, ‘B’ stands for ’bracket’ ,’O’for ‘of’ , ‘D’ for’ division’ and ‘M’ for
‘multiplication’, ‘A’ for ‘addition’ and ‘S’ for ‘subtraction’.
Thus, in simplifying an expression, first of all the brackets must be removed, strictly
in the order(), {} and [].

After removing the brackets, we must use the following operations strictly in the
order: (1)of (2)division (3) multiplication (4)addition (5)subtraction.

II. Modulus of a real number : Modulus of a real number a is defined as


|a| ={a, if a>0
-a, if a<0
Thus, |5|=5 and |-5|=-(-5)=5.

III. Virnaculum (or bar): When an expression contains Virnaculum, before applying
the ‘BODMAS’ rule, we simplify the expression under the Virnaculum.

MCQ QUESTIONS FOR PRACTICE

1. What are the coefficients of x in the expression 8 – x + y?


(a) 1 (b) –1 (c) 8 (d) none of these

2. What are the coefficients of y in the expression 4x – 3y?


(a) 4 (b) –3 (c) 3 (d) none of these

Prepared by: M. S. KumarSwamy, TGT(Maths) Page - 14 -


3. What are the coefficients of y in the expression yz2 + 5?
(a) 5 (b) z (c) z2 (d) none of these

4. Write the expression for the statement: the sum of three times x and 11
(a) x +3 +11 ( b) 3x + 11 (c) 3 + 11x (d) 3x – 11

5. Write an expression : Raju s father s age is 5 years more than 3 times Raju s age .
If Raju s age is x years , then father’s age is
(a) 3x+5 (b) 5-3x (c) 3x-5 (d) 15x

6. Identify the coefficient of x in expression 8 – x + y


(a ) 0 (b) 8 (c) –1 (d) 1

7. The number of terms in 4p2q – 3pq2+5 is


(a) 7 (b) 3 (c) 1 (d) 4

8. The expression for sum of numbers a and b subtracted from their product is
(a) a + b – ab (b) ab – a + b (c) ab – (a+b) (d) ab + a – b.

9. The sum of mn + 5 – 2 and mn+3 is


(a) 2mn + 3 (b) 6 ( c) 2mn + 8 (d) 2mn + 6.

10. What is the statement for the expression 3mn + 5


1
(a) 5 more than of product of m and n
3
(b) number 5 added to product of number m and n
(c) number 5 added to 3 times the product of m and n.
(d) 5 more than 3 times the product of the numbers m and n

11. The constant term in the expression 1 + x2 + x is


(a) 1 (b) 2 (c) x (d) x2

12. The coefficient of y3 in the expression y – y3 + y2 is


(a) 1 (b) y (c) –y3 (d) –1

13. The number of terms in the expression 1.2ab – 2.4 b + 3.6a is


(a) 1.2 (b) –2.4 (c) 3.6a (d) 3

14. What is the numerical coefficient of y2 in the expression 2x2y – 15xy2 +7y
(a) –15x (b) –15 (c) 2 (d) 7

15. The expression x + y – xy is


(a) Monomial (b) Binomial (c) Trinomial (d) Quadrinomial

16. The expression xyz is


(a) Monomial (b) Binomial (c) Trinomial (d) Zero polynomial

17. From the following expressions 10pq, 7p, 8q, -p2q2, -7pq, -23, ab,3a,b.The like
terms are
(a) 3,7p (b) 10 pq, –7pq (c) ab,3a,b (d)10pq,7p,8q

18. From the following expressing 3ab,a2,b2,a,5ab, –2ab,2a2 the three terms are

Prepared by: M. S. KumarSwamy, TGT(Maths) Page - 15 -


(a) 3ab,5ab, –2ab (b) a2,a,2a2 (c)3ab,a2,b2 (d)2a2,a2, a

19. Sum of 3m and 2n is


(a)5mn (b) 3m+2n (c) 5m (d) 5n

20. Sum of xy, x+y and y+ xy is


(a) 2xy +2x+y (b) 3xy+2y (c) 2xy +x+y (d) 2xy+x+2y

21. The value of 21b – 32 + 7b – 20b is


(a) 48b – 32 (b) –8b – 32 (c)8b – 32 (d)28b – 52

22. Subtract a – b from a + b the result is


(a) 2a + 2b (b) 2a (c) 2b (d) 2a – 2b

23. Subtracting –5y2 from y2 ,the result is


(a) –4y2 (b) 6y2 (c) 4 y2 (d) –6y2

24. The value of expression 5n – 2, when n= –2 is


(a) –12 (b) 8 (c) 1 (d) –8

25. The value of expression 7a – 4b for a = 3, b = 2 is


(a) 13 (b) 7a – 6b (c) 21a – 8b (d) 29

26. When x = 0, y = –1, then the value of expression 2x + 2y is


(a) 4 (b) 0 (c) –2 (d) 2

27. Factors of the term 15x2 in the expression 15x2 –13x are
(a) 15, x, x (b) 15,-13 (c) 15x2, –13x (d)15

28. Factors of the terms –4pq2 in the expression 9p2q2 –4 pq2 are
(a) 9 p2q2 , –4 pq2 (b)9, –4 (c) –4,p,q,q (d) –4

29. If the length of each side of the equilateral triangle is l, then the perimeter of the
equilateral triangle is
(a) 3l (b) 3+l (c) 3-l (d) l/3

30. Which of the following is monomial


(a) 2x +3 (b) 2x (c) 4x + 2y + 3 (d) 4y + 5x + z – 1

31. Which of the following is trinomial


(a) 2a + 6b – 1 (b) 1 (c) 5a – 7 (d) a + b + c – 3

32. Terms with factors y in the expression 8 + xy + xyz are


(a) xy, xyz (b) x, xz (c) 8, xy, xyz (d) y, xz

33. Identify the terms in the expression x + y + 1 which are not constant
(a) x,y,1 (b) x, y (c) x,1 (d) y,1

34. The value of expression 4x – 3 at x=2 is


(a) –4 (b) 5 (c) 4 (d) 2

35. The value of expression 5n2 + 5n – 2 for n = –2 is


(a) 13 (b) 3 (c) 8 (d) 12
Prepared by: M. S. KumarSwamy, TGT(Maths) Page - 16 -
36. The value of expression 2a2 + 2b2 – ab for a=2, b=1is
(a) 2 (b) 8 (c) 6 (d) 10

37. The value of x + 7 + 4(x – 5) for x=2


(a) –3 (b) 31 (c) 12 (d) 37

38. The value of expression 2a – 2b – 4 – 5 + a at a=1, b=-2


(a) 10 (b) –2 (c) 12 (d) –4

39. What must be subtracted from 2a + b to get 2a – b


(a) 2b (b) 4a (c) 0 (d) 4a+4b

40. What must be added to 3x + y to get 2x + 3y


(a) 5x + 4y (b) –x + 2y (c) x – 2y (d) x + 2y

41. Subtract a + 2b from sum of a – b and 2a+b


(a) 2a – 2b (b) 4a +2b (c) 2b (d) –2a +2b

42. On simplifying (a + b – 3) – (b – a +3) + (a – b + 3) the result is


(a) a – b + 3 (b) a – b – 3 (c) 3a – b – 3 (d) 3a+b+3

43. What should be value of ‘a’ if y2 + y – a equals to 3 for y=1


(a) –1 (b) –5 (c) 5 (d) 0

44. What is an expression for the statement: “p is multiplied by 16”


(a) 16p (b) p/16 (c) p+16 (d) p-16

45. The expression for the statement: “ y multiplied by 10 and then 7 added to
product”.
(a) 10 + y + 7 (b) 7y + 10 (c) 10y + 7 (d) 10y

46. What is the statement for the expression 2y – 9


(a) 2y subtracted from 9 (b) 9 subtracted from y and multiplied by 2
(c) 9 subtracted from 9 (d) thrice of y minus 9

47. Give expression for: “ 5 times of ‘y’ to which 3 is added”


5
(a) y +15 (b) 5y + 3 (c) +3 (d) 3y +5
y
48. The equation for the statement: one forth of a number minus 4 gives 4.
4 1 1
(a) 4x – 4 = 4 (b) –4=4 (c) x – 4 = 4 (d) x – 4 =
x 4 4

49. The area of triangle is’ xy’ where’ x’ is length and ‘y’ is breadth. If the length of
rectangle is increased by 5 units and breadth is decreased by 3 units, the new area
of rectangle will be
(a) (x – y)(x + 3) (b) xy+15 (c) (x + 5)(y – 3) (d) xy + 5 – 3

50. Area of rectangle of length’ 3x’ and breadth ‘5y ‘is


(a) 3x + 5y (b) 15xy (c)15x (d)15y

Prepared by: M. S. KumarSwamy, TGT(Maths) Page - 17 -


SQUARE ROOTS AND CUBE ROOTS

IMPORTANT FACTS AND FORMULAE


Square Root: If x2 = y, we say that the square root of y is x and we write, y  x.

Thus, 4  2, 9  3, 196  14, .


Cube Root: The cube root of a given number x is the number whose cube is x. We
denote the cube root of x by 3 x .
Thus, 3 8  3 2  2  2  2, 3 343  3 7  7  7  7 etc.
x x
Note: 1. xy  x  y 2. 
y y

MCQ QUESTIONS FOR PRACTICE

1. Which is the smallest three-digit perfect square?


(a) 100 (b) 101 (c) 121 (d) 144

2. Which is the greatest three-digit perfect square?


(a) 999 (b) 961 (c) 962 (d) 970

3. Which is the greatest 4-digit perfect square?


(a) 9999 (b) 9990 (c) 9800 (d) 9801

4. Which is the smallest 4-digit perfect square?


(a) 1024 (b) 1025 (c) 1000 (d) 1016

5. What will be the number of digits in the square root of 25600?


(a) 3 (b) 2 (c) 5 (d) 4

6. What will be the number of digits in the square root of 1296?


(a) 2 (b) 3 (c) 1 (d) 4

7. The square root of 12.25 is _______________ .


(a) 3.5 (b) 2.5 (c) 35 (d) 25

8. What is the length of the side of a square whose area is 441 cm2 ?
(a) 21 (b) 22 (c) 20 (d) 12

9. In a right angle triangle ABC , right angled at B, AB=6cm, BC=8cm ,then AC=
____ .
(a) 10 (b) 12 (c) 21 (d) 14

10. Which least number should be subtracted from 629 so as to get a perfect square ?
(a) 4 (b) 5 (c) 6 (d) 3

11. The square root of 1.21 is


(a) 1.1 (b) 11 (c) 21 (d) 2.1

Prepared by: M. S. KumarSwamy, TGT(Maths) Page - 18 -


12. What is the smallest square number which is divisible by each of the numbers 6,9
and 15 ?
(a) 900 (b) 810 (c) 630 (d) 720

13. The square of 1.2 is


(a) 144 (b) 1.44 (c) 14.4 (d) 2.4

14. The square root of 169 is


13
(a) -13 (b) 1.3 (c) -1.3 (d)
10

15. What is the length of the diagonal of a rectangle having dimensions 3cm and 4cm?
(a) 5 (b) 7 (c) 1 (d) 4

16. What will be the length of third side of a right angled triangle whose hypotenuse
is 5cm and one of the side is 3 cm ?
(a) 2 (b) 3 (c) 4 (d) 5

17. Which of the following is not a perfect square ?


(a) 81 (b) 18 (c) 100 (d) 121

18. Which is the smallest square number that is divisible by each of the number 4,9
and10?
(a) 900 (b) 810 (c) 800 (d) 920

19. Which of the following is not a square number?


( a) 4 (b)9 ( c) 16 ( d ) 24

20. The square of 23 is :


( a ) 529 ( b ) 526 ( c ) 46 ( d ) 429

21. The square of which of the following would be even number?


(a ) 2826 ( b ) 7779 ( c ) 1057 ( d ) 131

22. The square of which of the following would be odd number?


( a) 431 ( b) 272 ( c) 1234 ( d ) 7928

23. Which of the following is a perfect square ?


( a ) 45 ( b) 81 ( c ) 18 ( d ) 54

24. What will be the “ one’s digit” in the square of 1234 ?


(a)6 ( b) 2 (c)8 (d)9

25. What will be the number of zeros in the square of 400 ?


( a) 5 ( b) 1 (c)3 (d)4

26. The perfect square number between 30 and 40 is :


( a) 35 ( b ) 39 ( c ) 36 ( d ) 32

27. Which of the following number would have digit 6 at units place ?
( a ) 192 ( b ) 242 ( c) 252 ( d) 132

Prepared by: M. S. KumarSwamy, TGT(Maths) Page - 19 -


28. Which of the following number would have digit 5 at units place :
( a ) 952 ( b) 592 ( c ) 242 (d) 422

29. Which of the following number would have digit 1 at units place ?
( a ) 812 ( b ) 182 ( c ) 542 ( d ) 952

30. How many natural numbers lie between 92 and 102?


(a)15 (b) 19 (c) 18 (d) 17

31. How many non square numbers lie between 112and 122?
(a) 21 (b) 23 (c) 22 (d) 20
32. 25 can be express as the sum of first _________consecutive odd numbers .
(a) (b) 4 (c) 6 (d) 3
33. How many numbers lie between square of 12 and 13?
(a) 21 (b) 23 (c)22 (d)24
34. What will be the value of ‘ x’ in Pythagorean triplet (6,8, x)?
(a) 5 (b) 7 (c)10 (d) 11
35. The square of -9 is
(a) -81 (b) 81 (c) 18 (d) -18
36. The square root of 6400 is
(a) 80 (b) 81 (c) 32 (d) 23
37. By which smallest number 90 must be multiplied so as to make it a perfect square
?
(a) 10 (b) 2 (c) 5 (d) 3
38. By which smallest number 48 must be divided so as to make it a perfect square ?
(a) 2 (b) 3 (c) 6 (d) 4
39. Which smallest number should be added to 80 so as to make it a perfect square ?
(a) 2 (b) 3 (c) 1 (d) 4
40. What could be the possible “one’s digit” of the square root of 625?
(a) 5 (b) 0 (c) 4 (d) 8
41. The Smallest number by which 12348 must be divided to obtain a perfect square
is
(a) 3 (b) 5 (c) 4 (d) 7

42. 0.9 = ?
(a) 3 (b) 0.3 (c) 0.03 (d) 0.33

43. 1.0816 = ?
(a) 1.04 (b) 1.286 (c) 0.904 (d) 1.35

288
44. =?
128
1 3 5 9
(a) 2 (b) 2 (c) 2 (d) 2
14 14 14 14

Prepared by: M. S. KumarSwamy, TGT(Maths) Page - 20 -


45. 0.9  1.6 = ?
(a) 0.12 (b) 1.2 (c) 0.75 (d) 12

46. Which is the smallest three-digit perfect cube?


(a) 125 (b) 343 (c) 729 (d) 512

47. Which is the greatest three-digit perfect cube?


(a) 125 (b) 343 (c) 729 (d) 512

48. Which of the following is not a perfect cube ?


(a) 1 (b) 9 (c) 8 (d) 27

49. The cube of 4 is _______________ .


(a) 12 (b) 8 (c) 4 (d) 64

50. The value of 53 is __________ .


(a) 125 (b) 15 (c) 10 (d) 75

51. The cube of an even number is always ____________ .


(a) odd number (b) even number (c) prime number (d) none of these

52. The cube of an odd number is always __________ .


(a) odd number (b) even number (c) prime number (d) none of these

53. Each prime factor appears _________ times in its cube?


(a) 2 (b) 3 (c) 1 (d) 4

54. Which of the following is Hardy-Ramanujan Number ?


(a) 1724 (b)1725 (c) 1727 (d) 1729

55. By which smallest natural number 392 must be multiplied so as to make the
product a perfect cube ?
(a) 2 (b) 14 (c) 7 (d) 49

56. The smallest natural number by which 243 must be multiplied to make the
product a perfect cube is __________ .
(a) 3 (b) 9 (c) 8 (d) 7

57. The smallest natural number by which 704 must be divided to obtain a perfect
cube is
(a) 22 (b) 12 (c) 11 (d) 13

58. The smallest natural number by which 135 must be divided to obtain a perfect
cube is
(a) 5 (b) 3 (c) 15 (d) 9

59. Which of the following is not a perfect cube ?


(a) 216 (b) 343 (c) 125 (d) 108
60. The expansion of a3 is ___________ .
(a) 3 × a (b) a+a+a (c) 3 × 3 × 3 (d) a × a × a

Prepared by: M. S. KumarSwamy, TGT(Maths) Page - 21 -


61. What will be the unit digit of the cube of a number ending with 2 ?
(a) 8 (b) 4 (c) 2 (d) 6

62. What will be the unit digit of the cube of a number ending with 4 ?
(a) 4 (b) 6 (c) 2 (d) 8

63. What will be the unit digit of the cube of a number ending with 6 ?
(a) 4 (b) 6 (c) 2 (d) 8

64. A cuboid has dimensions 5cm , 2cm, 5cm .How many such cuboid will be needed
to form a cube ?
(a) 20 (b) 10 (c) 5 (d) 2

65. How many cuboids of dimensions 15cm, 30cm ,15cm will be needed to form a
cube ?
(a) 15 (b) 4 (c) 30 (d) 5

66. 729 is the value of _______________ .


(a) 83 (b) 93 (c) 63 (d) 43

67. Which of the following is a perfect cube ?


(a) 125 (b) 135 (c) 145 (d) 115

68. What is the volume of a cube whose edge is 2cm ?


(a) 8 (b) 6 (c) 10 (d) 4

69. The symbol for cube root is __________ .


(a) (b) 3 (c) 4
(d) none of these

70. The cube root of 512 is ________ .


(a) 8 (b) 32 (c) 16 (d) 2

3
71. The value of 343 is ____ .
(a) 8 (b) 7 (c) 6 (d) 3

72. Which of the following is true for any natural number n?


(a) n2>n3 (b) n3>n2 (c) n2=n3 (d) none of these

73. If the volume of a cube is 125 cm3 then what would be the length of its side?
(a) 25 (b) 5 (c) 4 (d) 15

74. What will be the unit digit of the cube root of a number ends with 8?
(a) 2 (b) 8 (c) 4 (d) 6

75. What will be the unit digit of the cube root of a number ends with 2?
(a) 2 (b) 8 (c) 4 (d) 6

76. What will be the unit digit of the cube root of a number ends with 3?
(a) 3 (b) 7 (c) 5 (d) 2

77. What will be the unit digit of the cube root of a number ends with 7?
(a) 3 (b) 7 (c) 6 (d) 5
Prepared by: M. S. KumarSwamy, TGT(Maths) Page - 22 -
78. 9 is the cube root of __________ .
(a) 343 (b) 729 (c) 629 (d) 81

79. The number of digits in the cube root of a 6-digit number is _______ .
(a) 3 (b) 2 (c) 4 (d) 6

80. How many digits will be there in the cube root of 46656 ?
(a) 2 (b) 1 (c) 3 (d) 4

81. How many digits will be there in the cube root of 512 ?
(a) 1 (b) 2 (c) 3 (d) 4

82. What will be the unit digit of 3 15625 ?


(a) 5 (b) 0 (c) 3 (d) 4

83. How many zeros will be there in the cube root of 27000?
(a) 3 (b) 0 (c) 1 (d) 2

84. How many zeros will be there in the cube root of 800?
(a) 3 (b) 0 (c) 1 (d) cube root does not
exist

85. If 73=343 , then 3


343 = _________ .
(a) 3 (b) 7 (c) 13 (d) 9

86. If 83=512 , then 3


512 = __________ .
(a) 3 (b) 7 (c) 13 (d) 9

87. What will be the unit digit of 3 216 ?


(a) 3 (b) 6 (c) 4 (d) 2

88. Find the one’s digit of the cube of 149


(a) 2 (b) 3 (c) 9 (d) none of these

89. Find the cube root of 8000.


(a) 20 (b) 200 (c) 40 (d) none of these

90. Find the cube root of 0.0027.


(a) 3 (b) 0.3 (c) 0.03 (d) none of these

AVERAGE
Sum of all the numbers
CONCEPT: Average of numbers =
Total numbers
MCQ QUESTIONS FOR PRACTICE
1. Average of first five multiples of 3 is ?
a) 11 b) 15 c) 9 d) 13
Prepared by: M. S. KumarSwamy, TGT(Maths) Page - 23 -
2. In Arun’s opinion, his weight is greater than 65 kg but less than 72 kg. His brother
doest not agree with Arun and he thinks that Arun’s weight is greater than 60 kg
but less than 70 kg. His mother’s view is that his weight cannot be greater than 68
kg. If all are them are correct in their estimation, what is the average of different
probable weights of Arun
a) 68 kg b) 67 kg c) 69 kg d) data inadequate

3. A library has an average of 510 visitors on Sundays and 240 on other days. The
average number of visitors per day in a month of 30 days beginning with a Sunday
is:
a) 250 b) 276 c) 280 d) 285

4. The average of 20 numbers is zero. Of them, at the most, how many may be
greater than zero?
a) 1 b) 19 c) 10 d) 0

5. Ages of ‘A’ and ‘B’ are in the ratio of 2 : 3 respectively. Six years hence the ratio
of their ages will become 8 : 11 respectively. What is B’s present age?
a) 27 yrs b) 28 yrs c) 18 yrs d) 25 yrs

6. The average weight of 16 boys in a class is 50.25 kg and that of the remaining 8
boys is 45.15 kg. Find the average weights of all the boys in the class.
a) 48.55 kg b) 49.25 kg c) 48 kg d) 47.55 kg

7. Sumitra has an average of 56% on her first 7 examinations. How much she should
make on her eighth examination to obtain an average of 60% on 8 examinations?
a) 92% b) 68% c) 88% d) 78%

8. The total age of A and B is 12 years more than that of total age of B and C. C is
how many years younger than A?
a) 12 b) 25 c) 26 d) C is elder than A

9. The average weight of A, B and C is 45 kg. If the average weight of A and B be


40 kg and that of B and C be 43 kg, then the weight of B is:
a) 20 kg b) 31 kg c) 17 kg d) 26 kg

10. A car owner buys petrol at Rs.7.50, Rs. 8 and Rs. 8.50 per litre for three
successive years. What approximately is the average cost per litre of petrol if he
spends Rs. 4000 each year?
a) Rs. 7.98 b) Rs. 8 c) Rs. 9 d) Rs. 8.50

11. The average weight of 8 persons increases by 2.5 kg when a new person comes in
place of one of them weighing 65 kg. What might be the weight of the new
person?
a) 80 kg b) 85 kg c) 82 kg d) 76.5 kg

12. If the average marks of three batches of 55, 60 and 45 students respectively is 50,
55, 60, then the average marks of all the students is:
a) 55 b) 53.33 c) 54.68 d) none of these

13. The average of five results is 46 and that of the first four is 45. The fifth result is ?
a) 12.5 b) 10 c) 1 d) 50

Prepared by: M. S. KumarSwamy, TGT(Maths) Page - 24 -


14. The average weight of 8 person’s increases by 2.5 kg when a new person comes in
place of one of them weighing 65 kg. What might be the weight of the new
person?
a) Data inadequate b) 85 kg c) 76 kg d) 76.5 kg

15. In a school with 600 students, the average age of the boys is 12 years and that of
the girls is 11 years. If the average of the school is 11 years 9 months, then the
number of girls in the school is: ?
a) 450 b) 250 c) 150 d) 350

16. A pupil’s marks were wrongly entered as 83 instead of 63. Due to that the average
marks for the class got increased by half (1/2). The number of pupils in the class
is:
a) 40 b) 20 c) 73 d) 10

17. A family consists of two grandparents, two parents and three grandchildren. The
average age of the grandparents is 67 years, that of the parents is 35 years and that
of the grandchildren is 6 years. What is the average age of the family?
5 1 4
a) none of these b) 31 yrs c) 32 yrs d) 28 yrs
7 7 7

18. In the first 10 overs of a cricket game, the run rate was only 3.2. What should be
the run rate in the remaining 40 overs to reach the target of 282 runs?
a) 6.25 b) 6.5 c) 6.75 d) 7

19. A grocer has a sale of Rs. 6435, Rs. 6927, Rs. 6855, Rs. 7230 and Rs. 6562 for 5
consecutive months. How much sale must he have in the sixth month so that he
gets an average sale of Rs. 6500?
a) Rs. 6001 b) Rs.5991 c) Rs.4991 d) Rs.6991

20. The average monthly income of P and Q is Rs. 5050. The average monthly
income of Q and R is Rs. 6250 and the average monthly income of P and R is Rs.
5200. The monthly income of P is:
a) 3500 b) 4500 c) 4000 d) 5000

21. The captain of a cricket team of 11 members is 26 years old and the wicket keeper
is 3 years older. If the ages of these two are excluded, the average age of the
remaining players is one year less than the average age of the whole team. What is
the average age of the team?
a) 23 yrs b) 24 yrs c) 25 yrs d) none of these

22. The average age of husband, wife and their child 3 years ago was 27 years and
that of wife and the child 5 years ago was 20 years. The present age of the
husband is:
a) none of these b) 35 yrs c) 40 yrs d) 50 yrs

23. The arithmetic mean of the scores of a group of students in a test was 52. The
brightest 20% of them secured a mean score of 80 and the dullest 25% a mean
score of 31. The mean score of remaining 55% is: ?
a) 45 b) 54.6 approx. c) 50 d) 51.4 approx.

Prepared by: M. S. KumarSwamy, TGT(Maths) Page - 25 -


24. The average of six numbers is 30. if the average of first four is 25 and that of last
three is 35, the fourth number is : ?
a) 35 b) 30 c) 25 d) 40

25. If the average marks of three batches of 55, 60 and 45 students respectively is 50,
55 and 60, then the average marks of all the students is: ?
a) none of these b) 54.68 c) 53.33 d) 55

26. The captain of a cricket team of 11 members is 26 years old and the wicket keeper
is 3 years older. If the ages of these two are excluded, the average age of the
remaining players is one year less than the average age of the whole team. What is
the average age of the team?
a) 23 yrs b) 24 yrs c) 25 yrs d) none of these

27. The average age of 24 boys and the teacher is 15 years. When the teaher’s age is
excluded, the agerage decreases by 1. What is the age of the teacher ?
a) 38 yrs b) 39 yrs c) 40 yrs d) none of these

28. In the first 10 overs of a cricket game, the run rate was only 3.2. What should be
the run rate in the remaining 40 overs to reach the target of 282 runs?
a) 6.75 b) 6.25 c) 6.5 d) 7

29. Out of 9 persons, 8 persons spent Rs. 30 each for their meals. The ninth one spent
Rs. 20 more than the average expenditure of all the nine. The total money spent by
all of them was: ?
a) Rs. 260 b) Rs.290 c) Rs.292.50 d) Rs.400.50

30. The average age of students of a class is 15.8 years. The average age of boys in
the class is 16.4 years and that of the girls is 15.4 years. The ratio of the number of
boys to the number of girls in the class is: ?
a) 3 : 5 b) 1 : 2 c) 2 : 3 d) 3 : 4

31. The average score of a cricketer for ten matches is 38.9 runs. If the average for the
first six matches is 42. Then find the average for the last four matches?
a) 33.25 b) 33.5 c) 35 d) 34.25

32. Average of all prime numbers between 30 to 50 ?


a) 37.8 b) 39 c) 37 d) 39.8

33. A car owner buys petrol at Rs 7.50, Rs. 8 and Rs. 8.50 per litre for three
successive years. What approximately is the average cost per litre of petrol if he
spends Rs. 4000 each year?
a) Rs. 8.50 b) Rs.7.98 c) Rs.8 d) Rs.9

34. A cricketer has a certain average for 9 innings. In the tenth inning He scores 100
runs. thereby increasing his average by 8 runs. His new average is : ?
a) 24 runs b) 32 runs c) 20 runs d) 28 runs

35. After replaceing an old member by a new member, It was found that the average
age of five members of a club is the same as it was 3 years ago. What is the
difference between the ages on the replaed and the new member ?
a) 8 yrs b) 15 yrs c) 4 yrs d) 2 yrs

Prepared by: M. S. KumarSwamy, TGT(Maths) Page - 26 -


36. Average of 10 matches is 32, How many runs one should should score to increase
his average by 4 runs.?
a) 76 b) 70 c) 78 d) 80

37. When a student weighing 45 kgs left a class, the average weight of the remaining
59 students increased by 200g. What is the average weight of the remaining 59
students ?
a) 55 b) 56 c) 57 d) 58

38. Distance between two stations A and B is 778 km. A train covers the journey from
A to B at 84 km per hour and returns back to A with a uniform speed of 56km per
hour. Find the average speed of the train during the whole journey?
a) 67.2 km/hr b) 67.0 km/hr c) 69.0 km/hr d) 69.2 km/hr

39. The average of 11 observations is 60. If the average of first five observations is 58
and that of the last five is 56, then the sixth observation is : ?
a) 90 b) 110 c) 85 d) 100

40. There are two sections A and B of a class, consisting of 36 and 44 students
respectively. If the average weight of sections A is 40 kg and that of section b is
35 kg. Find the average weight of the whole class?
a) 39.25 b) 36.25 c) 37.25 d) 38.25

SURDS AND INDICES


I IMPORTANT FACTS AND FORMULAE I
1. LAWS OF INDICES:
(i) am x an = am + n
(ii) am / an = am-n
(iii) (am)n = amn
(iv) (ab)n = anbn
n
(v) a an
  
b bn
0
(vi) a =1
1
(vii) a n  n
a

2. SURDS: Let a be a rational number and n be a positive integer such that a1/n = n
a
n
is irrational. Then a is called a surd of order n.

3. LAWS OF SURDS:

(i ) a  a 1 / 2
(ii ) n a  a 1/ n
(iii ) n ab  n
a .n b
n
a a
(iv )n  n
b b
Prepared by: M. S. KumarSwamy, TGT(Maths) Page - 27 -
n
(v )  a
n
a

(vi ) m n a  mn a
m
(vii )  an
 n am

MCQ QUESTIONS FOR PRACTICE


2/ 5
1. The value of 32 is :
(a) 2 (b) 4 (c) 16 (d) 14

3/ 4
2. The value of 16 is :
(a) 4 (b) 12 (c) 8 (d) 48

1
3
3. The value of 125 is :
1 1 1 1
(a) (b) (c) (d)
5 25 15 125

4. The value of 1 11 / 2  1 11 / 4 is :
(a) 111/ 4 (b) 113/ 4 (c) 111/8 (d) 111/ 2

5. The value of 6 4  3 / 2 is :
1 1 1
(a) (b) (c) 512 (d)
96 64 512

2
6. The value of 125  3 is :
(a) 5 (b) 25 (c) 45 (d) 35

3/ 2
7. The value of 25 is :
(a) 5 (b) 25 (c) 125 (d) 625

1
2
8. The value of 64 is :
(a) 8 (b) 4 (c) 16 (d) 32

1
9. The value of 32 5 is :
(a) 16 (b) 160 (c) 2 (d) 18

1
10. The value of 125  3 is :
(a) 5 (b) 25 (c) 45 (d) 35
3
2
11. The value of 9 is :
1
(a) 18 (b) 27 (c) – 18 (d)
27
Prepared by: M. S. KumarSwamy, TGT(Maths) Page - 28 -
12. For Positive real numbers a and b, which is not true?
(a) ab  a b  
(b) a  b a  b  a 2  b 
a a
(c)  (d) 
a b 
a  b  ab 
b b
13. Out of the following, the irrational number is
(a) 1.5 (b) 2.477 (c) 1.277 (d) 
1
14. To rationalize the denominator of ,we multiply this by
a b
1 1 a b a b
(a) (b) (c) (d)
a b a b a b a b

15. The number of rational numbers between 3 and 5 is


(a) One (b) 3 (c) none (d) infinitely many

16. If we add two irrational numbers, the resulting number


(a) is always an irrational number
(b) is always a rational number
(c) may be a rational or an irrational number
(d) always an integer

17. The rationalizing factor of 7  2 3 is


(a) 7  2 3 (b) 7  2 3 (c) 5  2 3 (d) 4  2 3

1 4
18. If  0.142857 , then equals
7 7
(a) 0.428571 (b) 0.571428 (c) 0.857142 (d) 0.285718

19. The value of n for which n be a rational number is


(a) 2 (b) 4 (c) 3 (d) 5

3 12
20. equals
6 27
1 1
(a) (b) 2 (c) 3 (d)
2 3

 
21. 3  3 3  2 equals
(a) 9  5 2  6 (b) 9  6 (c) 3  2 (d) 9  3 2  3 3  6

22. The arrangement of 2, 5, 3 in ascending order is


(a) 2, 3, 5 (b) 2, 5, 3 (c) 5, 3, 2 (d) 3, 2, 5

23. If m and n are two natural numbers and mn = 32, then nmn is
(a) 52 (b) 53 (c) 510 (d) 512

Prepared by: M. S. KumarSwamy, TGT(Maths) Page - 29 -


1
24. If 10  3.162 , then the value of is
10
(a) 0.3162 (b) 3.162 (c) 31.62 (d) 316.2

6 5 x 2
 3   16   4 
25. If         , then the value of x is
4  9   3
(a) 2 (b) 4 (c) -2 (d) 6

PRACTICE QUESTIONS

1. Evaluate the following expressions:


3 1
1
 256  8  343  3
(i )   (ii ) 15625  6 (iii )  
 6561   1331 
1
6561 
3
(iv) 8 (v)343
65536
32  48
2. Simplify:
8  12
7
3. Simplify:
3 32 2

4 3
4. Simplify: (i) 22 (ii ) 3 2. 4 2.12 32

5. Evaluate the following expressions:


1
 2 1 4 3
 625  4 
(i )  
3
(ii)27  27  27 3 3
(iii)  6.25  2
 81 
5 1
(iv)  0.000064  6

(v) 17  8 2

2 2

3
(4 2 )
6. Simplify:  2 5 6 
3 5 1
7. If a  then find the value of a 2  2 .
2 a
8. Simplify:
1
  1 1 34

(i) 5  8 3  27 3   (ii) 45  3 20  4 5
   
24 54
(iii )  (iv) 4 12  6 7 (v) 4 28  3 7
8 9

Prepared by: M. S. KumarSwamy, TGT(Maths) Page - 30 -


1 2
(i ) 3 3  2 27 
3
(ii )  3 5 
9. Simplify: (iii ) 4 81  8 3 216  15 5 32  225
2
3 1 3 3
(iv )  ( v) 
8 2 3 6

4 1 2
10. Find the value of 2
 3
 1
 216  3  256  4  243 5
1 1
3 2
9  27
11. Simplify: 1 2
6 3
3 3
12. Simplify:
2
1 4 12 6
3 8  32   1  3

(i ) 13  23  3 
3 2
(ii)    
5 5
 
 5 
(iii )   
 27 
2 1 1
1
 1 4  3 3  13 1
2

  8 16
(iv)   625  2   (v ) 1
(vi )64 64  64 3 
3
   3  
  32

13. Find the value of a and b in each of the following:


3 2 3 7 7 5
(i )  ab 2 (ii )  ab 7 (iii )  ab 5
3 2 3 7 7 5

14. Simplify each of the following by rationalizing the denominator.


64 2 5 2 52
(i ) (ii ) 
64 2 52 5 2

3 2 1 1
15. If x = , find (i) x 2  2 (ii) x 4  4 .
3 2 x x

PERCENTAGE, PROFIT & LOSS & DISCOUNT

IMPORTANT FACTS AND FORMULAE

1. Concept of Percentage : By a certain percent ,we mean that many hundredths.


Thus x percent means x hundredths, written as x%.
x
To express x% as a fraction : We have , x% = .
100

Prepared by: M. S. KumarSwamy, TGT(Maths) Page - 31 -


20 1 48 12
Thus, 20% =  ; 48% =  , etc.
100 5 100 25
a a
To express a/b as a percent : We have, =  100% .
b b
1 1
Thus,   100%  25%
4 4
2. If the price of a commodity increases by R%, then the reduction in consumption
R
so as not to increase the expenditure is  100%
100  R
If the price of the commodity decreases by R%,then the increase in consumption
R
so as to decrease the expenditure is 100%
100  R
3. Results on Population : Let the population of the town be P now and suppose it
increases at the rate of R% per annum, then :
n
  R 
1. Population after n years = P 1   
  100  
P
2. Population n years ago = n
  R 
1   100  
  
4. Results on Depreciation : Let the present value of a machine be P. Suppose it
depreciates at the rate R% per annum. Then,
n
  R 
1. Value of the machine after n years = P 1   
  100  
P
2. Value of the machine n years ago = n
  R 
1   100  
  

PROFIT AND LOSS


COST PRICE: The Price at which article is purchased. It is abbreviated as C.P.

SELLING PRICE: The price at which article is sold. It is abbreviated as S.P.

PROFIT OR GAIN: If SP is greater than CP, then the selling price is said to have a
profit or gain

LOSS: If SP is less than CP, then the selling price is said to have a loss.

Prepared by: M. S. KumarSwamy, TGT(Maths) Page - 32 -


FORMULA
1.Gain  SP  CP
2.Loss  CP  SP
Gain
3.Gain%  100%
CP
Loss
4.Loss %  100%
CP
100  Gain%
5.SP   CP
100
100  Loss%
6.SP   CP
100
100
7.CP   SP
100  Gain%
100
8.CP   SP
100  Loss %

Discount %
Discount =  M .P.
100
S.P. = M.P. – Discount
100
M .P.   SP
100  Discount %

MCQ QUESTIONS FOR PRACTICE

1. 72% of 25 students are good in hindi, how many are not good in hindi?
a) 16 b) 14 c) 18 d) 7
2. In a computer lab, there are 3 computers for every 6 students. How many
computers will be needed for 24 students?
a) 12 b) 14 c) 16 d) none of these
3. Out of 32 students, 8 are absent. What percent of the students are present?
a) 75% b) 64% c) 60% d) none of these
4. There are 25 radios, 16 of them are out of order. What percent of radios are out of
order?
a) 75% b) 64% c) 60% d) none of these
5. A shop has 500 parts, out of which 5 are defective. What percent are not
defective?
a) 75% b) 99% c) 90% d) none of these
6. There are 120 voters, 90 of them voted yes. What percent voted yes?
a) 75% b) 99% c) 90% d) none of these

7. The population of a town increased from 1,75,000 to 2,62,500 in a decade. The


average percent increase of population per year is:
a) 7% b) 5% c) 9% d) 8.75%
8. A survey of 40 children showed that 25% liked playing football. How many
children not liked playing football?
a) 90 b) 60 c) 30 d) none of these
Prepared by: M. S. KumarSwamy, TGT(Maths) Page - 33 -
9. When 15% is lost in grinding wheat, a country can export 30 lakh tons of wheat.
On the other hand, if 10% is lost in grinding, it can export 40 lakh tons of wheat.
The production of wheat in the country is:
(a) 40 lakh tons (b) 400 lakh tons
(c) 200 lakh tons (d) 900 lakh tons

10. In a History examination, the average for the entire class was 80 marks. If 10% of
the students scored 95 marks and 20% scored 90 marks, what was the average
marks of the remaining students of the class?
(a) 60 (b) 72 (c) 75 (d) 85

11. A’s marks in Biology are 20 less than 25% of the total marks obtained by him in
Biology, Maths and Drawing. If his marks in Drawing be 50,what are his marks in
Maths?
(a) 60 (b) 47 (c) 63 (d) cannot be determined

12. In an election between two candidates, one got 65%of the total valid votes, 20%
of the votes were invalid. If the total number of votes was 6500, the number of
valid votes that the other candidate got, was
(a) 1820 (b) 1840 (c) 1824 (d) 1844

13. 8% children of a class of 25 like getting wet in the rain. How many children do
not like getting wet in the rain.
a) 20 b) 22 c) 23 d) none of these

14. Rahul bought a sweater and saved Rs 20 when a discount of 25% was given. What
was the price of the sweater before the discount?
a) Rs 30 b) Rs 40 c) Rs 60 d) Rs 80

15. Out of 15,000 voters in a constituency, 60% voted. Find the number of voters who
did not vote.
a) 9000 b) 6000 c) 3000 d) none of these

16. Meeta saves Rs 400 from her salary. If this is 10% of her salary. What is her
salary?
a) 4000 b) 6000 c) 3000 d) none of these

17. A local cricket team played 20 matches in one season. It won 25% of them. How
many matches did they lose?
a) 12 b) 14 c) 16 d) none of these

18. A school team won 6 games this year against 4 games won last year. What is the
per cent increase?
a) 75% b) 50% c) 60% d) none of these

19. The number of illiterate persons in a country decreased from 150 lakhs to 100
lakhs in 10 years. What is the percentage of decrease?
1
a) 30% b) 50% c) 33 % d) none of these
3

20. Cost of an item is Rs 50. It was sold with a profit of 12%. Find the selling price.
a) Rs 56 b) Rs 60 c) Rs 70 d) none of these

Prepared by: M. S. KumarSwamy, TGT(Maths) Page - 34 -


21. How much will an item cost if 10% discount is given on the marked price Rs 100
a) 90 b) 110 c) 95 d) 85

22. A football team won 10 matches out of the total number of matches they played. If
their win percentage was 40, then how many matches did they play in all?
a) 10 b) 25 c) 40 d) none of these

23. If Chameli had Rs 600 left after spending 75% of her money, how much did she
have in the beginning?
a) Rs 3000 b) Rs 2400 c) Rs 2600 d) Rs 2800

24. The price of a scooter was Rs 34,000 last year. It has increased by 20% this year.
What is the price now?
a) Rs 40800 b) Rs 32300 c) Rs 40000 d) none of these

25. The price of a scooter was Rs 34,000 last year. It has decreased by 5% this year.
What is the price now?
a) Rs 40800 b) Rs 32300 c) Rs 40000 d) none of these

26. Mohit bought a CD for Rs. 750 and sold it Rs. 875. Find his gain or loss percent.
2
a) 5% b) 16% c) 6% d) 16 %
3
27. Rahul purchased a table for Rs. Rs. 1260 and due to some scratches on its top he
had to sell it for Rs. 1197. Find his loss or gain percent.
2
a) 5% b) 4% c) 6% d) 16 %
3
28. Raghu bought an almirah for Rs. 6250 and spent Rs. 375 on its repairs. Then he
sold it for Rs. Rs. 6890. Find his gain or loss percent.
2
a) 5% b) 4% c) 6% d) 16 %
3

29. A vendor bought 20 oranges for Rs. 56 and sold them at Rs. 35 per dozen. Find
his gain or loss percent.
1 2
a) 5% b) 4 % c) 6% d) 16 %
6 3

30. The cost of a flower vase is Rs 120. If the shopkeeper sells it at a loss of 10%, find
the price at which it is sold.
a) Rs 108 b) Rs 450 c) Rs 160 d) none of these

31. Selling price of a toy car is Rs 540. If the profit made by shopkeeper is 20%, what
is the cost price of this toy?
a) Rs 108 b) Rs 450 c) Rs 160 d) none of these

32. The marked price of a ceiling fan is Rs. 1250 and the shopkeeper allows a
discount of 6% on it. Find the selling price of the fan.
a) Rs 1180 b) Rs 1175 c) Rs 1160 d) none of these

Prepared by: M. S. KumarSwamy, TGT(Maths) Page - 35 -


33. A trader marks his goods at 40% above the cost price and allows a discount of
25%. What is his gain?
a) Rs 118 b) Rs 175 c) Rs 105 d) none of these

34. A dealer purchased a washing machine for Rs. 7660. He allows a discount of 12%
on its marked price and still gains 10%. Find the marked price of the machine.
a) Rs 9800 b) Rs 9675 c) Rs 9575 d) none of these
35. On selling a fan for Rs. 810. Sunil gains 8%. For how much did he purchase it?
a) Rs 780 b) Rs 750 c) Rs 760 d) none of these

36. On selling a table for Rs. 987. Ramesh loses 6%. For how much did he purchase
it?
a) Rs 1500 b) Rs 105 c) Rs 1050 d) none of these

2
37. Rashmi buys a calculator for Rs. 720 and sells it at a loss of 6 %. For how much
3
does she sell it?
a) Rs 700 b) Rs 650 c) Rs 672 d) none of these

38. The difference of two numbers is 20% of the larger number. If the smaller number
is 20, then the larger number is:
(a) 25 (b) 46 (c) 27 (d) 82

39. When any number is divided by 12, then dividend becomes 1/4th of the other
number. By how much percent first number is greater than the second number?
(a) 165 (b) 200 (c) 300 (d) 400

40. If one number is 80% of the other and 4 times the sum of their squares is 656, then
the numbers are:
(a) 6,8 (b) 8, 10 (c) 16, 20 d) 10, 15

41. Two numbers A and B are such that the sum of 5% of A and 4% of B is two-third
of the sum of 6% of A and 8% of B. Find the ratio of A : B.
(a) 1 : 2 (b) 3 : 1 (c) 3 : 4 (d) 4 : 3

42. Three candidates contested an election and received 1136, 7636 and 11628 votes
respectively. What percentage of the total votes did the winning candidate get?
(a) 57% (b) 77% (c) 80% (d) 90%

43. The population of a town increased from 1,75,000 to 2,62,500 in a decade. The
average percent increase of population per year is:
(a) 7% (b) 5% (c) 9% (d) 8.75%

44. A student multiplied a number by 3 instead of 5/3. What is the percentage error in
the calculation?
(a) 36% (b) 44% (c) 55% (d) 35%

45. A tempo is insured to the extent of 4/5 of its original value. If the premium on it at
the rate of 1.3 percent amounts to Rs. 910, the original value of the tempo is:
(a) Rs.78,000 (b) Rs.78,500 (c) Rs.80,000 (d) Rs.87,500

Prepared by: M. S. KumarSwamy, TGT(Maths) Page - 36 -


RATIO AND PROPORTION
IMPORTANT FACTS AND FORMULAE
I. RATIO: The ratio of two quantities a and b in the same units, is the fraction a/b and we
write it as a:b.
In the ratio a:b, we call a as the first term or antecedent and b, the second term or
consequent.

Ex. The ratio 5: 9 represents 5/9 with antecedent = 5, consequent = 9.


Rule: The multiplication or division of each term of a ratio by the same non-zero
number does not affect the ratio.
Ex. 4: 5 = 8: 10 = 12: 15 etc. Also, 4: 6 = 2: 3.

2. PROPORTION: The equality of two ratios is called proportion.


If a: b = c: d, we write, a: b:: c : d and we say that a, b, c, d are in proportion . Here a
and d are called extremes, while b and c are called mean terms.
Product of means = Product of extremes.
Thus, a: b:: c : d <=> (b x c) = (a x d).

3. (i) Fourth Proportional: If a : b = c: d, then d is called the fourth proportional


to a, b, c.
(ii) Third Proportional: If a: b = b: c, then c is called the third proportional to
a and b.
(iii) Mean Proportional: Mean proportional between a and b is square root of ab

4. (i) COMPARISON OF RATIOS:


We say that (a: b) > (c: d) <=> (a/b)>(c /d).
(ii) COMPOUNDED RATIO:
The compounded ratio of the ratios (a: b), (c: d), (e : f) is (ace: bdf)

5. (i) Duplicate ratio of (a : b) is (a2 : b2).


(ii) Sub-duplicate ratio of (a : b) is (√a : √b).
(iii)Triplicate ratio of (a : b) is (a3 : b3).
(iv) Sub-triplicate ratio of (a : b) is (a ⅓ : b ⅓ ).
(v) If (a/b)=(c/d), then ((a+b)/(a-b))=((c+d)/(c-d)) (Componendo and dividendo)

6. VARIATION:
(i) We say that x is directly proportional to y, if x = ky for some constant k and
we write, x  y.
(ii) We say that x is inversely proportional to y, if xy = k for some constant k and
we write, x∞(1/y)

PARTNERSHIP
1. Partnership: When two or more than two persons run a business jointly, they are
called partners and the deal is known as partnership.
2. Ratio of Division of Gains:
i) When investments of all the partners are for the same time, the gain or loss
is distributed a among the partners in the ratio of their investments.
Suppose A and B invest Rs. x and Rs. y respectively for a year in a business, then at

Prepared by: M. S. KumarSwamy, TGT(Maths) Page - 37 -


the end of the year:
(A’s share of profit) : (B's share of profit) = x : y.

ii) When investments are for different time periods, then equivalent capitals are
calculated for a unit of time by taking (capital x number of units of time). Now, gain
or loss is divided in the ratio of these capitals.

Suppose A invests Rs. x for p months and B invests Rs. y for q months, then
(A’s share of profit) : (B's share of profit) = xp : yq.

3. Working and Sleeping Partners: A partner who manages the business is known .
as a working partner and the one who simply invests the money is a sleeping partner.

MCQ QUESTIONS FOR PRACTICE

1. The ratio of 90 cm to 1.5 m is...........


(a) 3 : 5 (b) 5 : 3 (c) 60 : 1 (d) 4:3

2. 6 : 4 is equivalent ratio of ..............


(a) 2 : 3 (b) 3 : 2 (c) 1:2 (d) 1 : 4

3. Find the ratio of 81 to 108 ?


(a) 3 : 4 (b) 5 : 9 (c) 4 : 3 (d) 9 : 20

15 ......
4. Fill in the blank :- 
18 6
(a) 5 (b) 4 (c) 3 (d) 7

5. Find the value of x in 4 : 3 = x : 12 ?


(a) 4 (b) 12 (c) 16 (d) 3

6. In proportion first and the last terms are called ______________________ .


(a) Mean terms (b) Extreme terms (c) Middle terms (d) None of these

7. The ratio is said to be in simplest form if common factor is _____________.


(a) 1 (b) 0 (c) -1 (d) None of these

8. Three terms a , b , c are said to be in proportion if ______________ .


(a) a : b = b : c (b) a : b = c : b (c) b : a = c : a (d) c : a = a : b

9. Four terms a , b , c , d are said to be in proportion if ___________________ .


(a) a : b = c : d (b) a : c = d : b (c) a : d = b : c (d) None of
these

10. If the cost of 6 cans of juice is Rs 210, then what is the cost of 4 cans of juice is
(a) Rs 120 (b) Rs 140 (c) Rs 100 (d) Rs 80

11. Fill in the blank :- 32 m : 64 m = 6 sec : ______


(a) 13 sec (b) 12 sec (c) 8 sec (d) 24sec

12. Which of the following is correct :-


(a) 3 : 4 = 15 : 25 (b) 12 : 24 = 6 : 12 (c) 7 : 3 = 14 : 3 (d) 5 : 10 = 9 : 20

Prepared by: M. S. KumarSwamy, TGT(Maths) Page - 38 -


13. The ratio of 15 Kg to 75 Kg is...........
(a) 1 : 5 (b) 5 : 1 (c) 3:5 (d) 15 : 3

14. 7 : 42 is equivalent ratio of ..............


(a) 7 : 6 (b) 6 : 1 (c) 1:6 (d) 6 : 7

15. Find the ratio of 33 Km to 121 Km ?


(a) 3 : 11 (b) 11 : 3 (c) 3 : 7 (d) 7 : 3

16. A stick 1.5m long casts a shadow 4.5m long at the same time when a pole casts a
shadow 9.6m long. Find the length of the pole
a) 3.4m b) 3.6m c) 3.2m d) 3.8m

17. Find three numbers in the 2 : 3: 5, the sum of whose squares is 342
a) 6, 8, 9 b) 6, 9 , 15 c) 4, 6, 10 d) 10, 15, 25

18. Three utensils contain equal mixtures of alcohol and water in the ratio 5 : 2, 3 : 1
and 6 : 1 respectively. If all the solutions are mixed together find the ratio of milk
and water in the final mixture.
a) 19 : 165 b) 65 : 19 c) 63 : 19 d) 67 : 19

1 1 1
19. Three friends divide Rs 1110/– among themselves in the , , . Find the largest
4 5 6
share?
a) Rs 460 b) Rs 470 c) Rs 450 d) Rs 490

20. The boys and girls in a school are into ratio 9 : 5. If the number of girls are 225
then the number of ways are
a) 415 b) 400 c) 405 d) 425

21. An alloy contains copper and zin (in the ration 9 : 5). If difference in the quantities
of two metals is 4000gms. What is the weight of the alloy in Quintals
a) 1.4 b) 1.6 c) 1.2 d) 1.8

22. The ratio between the rates of walking of X and Y is 3 : 4. if the time taken by Y
to cover a certain distance is 24 minutes, how much time is taken by X to cover
the same distance?
a) 36mts b) 40mts c) 32mts d) 48mts

23. If 4x + 3y : 6x + 5y = 11: 17 find x : y


a) 2 : 1 b) 3 : 1 c) 4 : 1 d) 5 : 1

24. If 4p2 + pq : 3pq – q2 = 12 : 5 find p : q


a) 4 : 5 or 3 : 4 b) 3 : 4 or 5 : 7 c) 5 : 6 or 3 : 4 d) None

25. The ages of P and Q are in the ratio 7 : 8, 6 years ago, their ages were in the ratio
5 : 6. Find their present ages
a) 21yrs; 28yrs b) 21yrs, 32yrs c) 21yrs; 24yrs d) 28yrs; 32yrs

26. The ratio between two numbers is 5 : 7 and their LCM is 210. Find the numbers
a) 20, 28 b) 30, 42 c) 40, 56 d) 60, 84

Prepared by: M. S. KumarSwamy, TGT(Maths) Page - 39 -


27. In a mixture of 84litres, the ratio of milk and water is 5 : 2. How much water must
be added to their mixture 2 : 1?
a) 12 litres b) 48litres c) 36litres d) 60litres

28. If p : q = 3 : 5 then 3p2 – 2pq + 5q2: p2 + 7pq – 2q2


a) 61 : 31 b) 61 : 33 c) 61 : 32 d) 61 : 42

29. Divide Rs 2490 among P, Q and R so that 4times P's share; 5times Q's share and
7times R's share may all be equal. What is the value of the difference in share
between P and R.
a) Rs 550 b) Rs 240 c) Rs 210 d) Rs 450

30. The cost of production of a mobile set is Rs 954. This cost has divided between
Material, Labour and Overheads in the ratio 3 : 4 : 2. Calculate the cost of
overhead expenses
a) Rs 222 b) Rs 202 c) Rs 212 d) Rs 242

2
31. What is the ratio whose terms differ by 40 and the measure of which is ?
7
16 16 16 8
a) b) c) d)
48 64 56 48

32. If 3m – 8 : 2m + 1 is the duplicate ratio of 2 : 3, find the value of m


a) 5 b) 4 c) 6 d) 3

33. If 3c – 9 : 5c + 4 is the triplicate ratio of 3 : 4, find the value of c


a) 14 b) 13 c) 16 d) 18

34. If 2K + 3 : 3k + 7 is the sub duplicate ratio of 9 : 25, find the value of k


a) 8 b) 9 c) 6 d) 7

35. Find the compound ratio of duplicate ratio of 5 : 6, the receprocal of 25 : 32 and
sub duplicate ratio of 81 : 64
a) 1 : 2 b) 2 : 1 c) 1 : 1 d) 3 : 2

a
36. If  1 then
b
ax ax a x
a) 1 b) 1 c) 1 d) all of the above
bx bx b x

a
37. If  1 then
b
ax a ax a a x a
a)  b)  c)  d) (1) and
bx b bx b b x b
(2)

a c e a  c  e......
38. If    k then
b d f b  d  f .....
1
a) 2k b) 3k c) k d)
k

Prepared by: M. S. KumarSwamy, TGT(Maths) Page - 40 -


39. If 20% of x is same as 40% of y, then x : y is equal to
a) 1 : 2 b) 1 : 3 c) 3 : 1 d) 2 : 1

40. If x : y = 2 : 3 and 2 : x = 1 : 2 then the value of y is


a) 8 b) 4 c) 6 d) 12

41. Two numbers are in the ratio 5 : 7 if each number is increased by 10 the ratio
becomes 3 : 4. The numbers are
a) 40, 56 b) 50, 84 c) 50, 70 d) 60, 84

42. What number must be added to each one of 6, 14, 18, 38 to make it equally
proportionate
a) 1 b) 2 c) 3 d) 4

43. The 4th proportional to 0.4, 0.12 and 0.3 is


a) 0.03 b) 0.3 c) 0.9 d) 0.09

44. The third proportional to 0.6 and 0.4 is


15 4 2
a) b) 3.75 c) d)
4 15 15

45. The mean proportional between 16 and 36 is


a) 28 b) 24 c) 26 d) 22

46. Two whole numbers whose sum is 64, cannot be in the ratio
a) 5 : 3 b) 7 : 1 c) 3 : 4 d) 9 : 7

47. The ratio of two numbers is 5 : 3 and their sum is 448. The greater than number is
a) 280 b) 270 c) 290 d) 300

48. If 25 : b = 3 : 9 then the value of b is


a) 75 b) 65 c) 85 d) 95

49. If a : b = 18 : 14 then a + b : a – b =
a) 8 : 2 b) 8 : 3 c) 8 : 1 d) 8 : 5
1 1
50. If A  B and B  C then C : A
3 2
a) 3 : 6 b) 1 : 6 c) 6 : 3 d) 6 : 1

1
51. The ratio which ( of Rs 6.30) bears to (0.4 of Rs 1.20) is
5
a) 21 : 8 b) 8 : 21 c) 21 : 7 d) 21 : 6
th th th
1 1 1
52. If of A = of B = of C then C : A : B is
4 6 8
a) 2 : 3 : 4 b) 3 : 2 : 4 c) 4 : 2 : 3 d) 2 : 4 : 3
1 1 1 1
53. If :  : then the value of y is
4 y y 0.81
a) 0.9 b) 0.27 c) 0.18 d) 1.8
Prepared by: M. S. KumarSwamy, TGT(Maths) Page - 41 -
1 3 5
54. A fractions bears the same ratio to as does to . The fraction is
27 7 9
7 1 45 5
a) b) c) d)
45 35 7 21

7 15 17
55. If A  , B  ,C  , the smallest ratio is
15 20 25
a) B b) C c) A d) None of these

DIRECT AND INDIRECT PROPORTION


_IMPORTANT FACTS AND FORMULAE
1. Direct Proportion: Two quantities are said to be directly proportional, if on the
increase (or decrease) of the one, the other increases (or decreases) to the same
Ex. 1. Cost is directly proportional to the number of articles.
(More Articles, More Cost)
Ex. 2. Work done is directly proportional to the number of men working on it
(More Men, More Work)

2. Indirect Proportion: Two quantities are said to be indirectly proportional,if on the


increase of the one, the other decreases to the same extent and vice-versa.
Ex. 1. The time taken by a car in covering a certain distance is inversely proportional
to the speed of the car.
(More speed, Less is the time taken to cover a distance)
Ex. 2. Time taken to finish a work is inversely proportional to the num of persons
working at it.
(More persons, Less is the time taken to finish a job)
Remark: In solving questions by chain rule, we compare every item with the term to
be found out.

TIME AND WORK


1. If A can do a piece of work in n days, then A's 1 day's work = (1/n).
2. If A’s 1 day's work = (1/n),then A can finish the work in n days.
3. A is thrice as good a workman as B, then:
Ratio of work done by A and B = 3 : 1.
Ratio of times taken by A and B to finish a work = 1 : 3.

MCQ QUESTIONS FOR PRACTICE

1. If the cost of 1 kg of sugar is Rs 18, then what would be the cost of 3 kg sugar?
(a) Rs. 54 (b) Rs. 6 (c) Rs. 18 (d) none of these

2. If the cost of 9 toys is Rs. 333, find the cost of 16 such toys.
(a) Rs. 594 (b) Rs. 596 (c) Rs. 592 (d) none of these

3. If 25 metres of cloth costs of Rs. 1575, how many metres of it can be bought for
Rs. 2016?
Prepared by: M. S. KumarSwamy, TGT(Maths) Page - 42 -
(a) 30 m (b) 32 m (c) 36 m (d) none of these

4. A worker is paid Rs. 1110 for 6 days. If his total wages during a month are Rs.
4625, for how many days did he work?
(a) 15 days (b) 25 days (c) 30 days (d) none of these

5. A car can cover a distance of 522 km on 36 litres of petrol. How far can it travel
on 14 litres of petrol?
(a) 230 km (b) 232 km (c) 203 km (d) none of these

6. If 13 metres of a uniform iron rod weighs 23.4 kg then what will be the weight of
6 metres of the same rod?
(a) 10 kg (b) 20 kg (c) 10.8 kg (d) none of these

7. The length of the shadow of a 3m high pole at a certain time of the day is 3.6 m.
What is the length of the height of another pole whose shadow at that time is 54 m
long?
(a) 30 m (b) 40 m (c) 45 m (d) none of these

8. Traveling 900 km by rail costs Rs. 280. What would be the fare for a journey of
360 km when a person travels by the same class?
(a) Rs. 118 (b) Rs. 112 (c) Rs. 119 (d) none of these

9. A train covers a distance of 51 km in 45 minutes. How long will it take to cover


221 km?
1 1
(a) 3 hours (b) 3 hrs (c) 3 hrs (d) none of these
4 2

10. If 15 oranges cost Rs. 70, what do 39 oranges cost?


(a) Rs. 180 (b) Rs. 182 (c) Rs. 190 (d) none of these

11. If 8 kg sugar costs Rs. 148, how much sugar can be bought for Rs. 832.50?
(a) 45 kg (b) 50 kg (c) 60 kg (d) none of these

12. The cost of 37m of silk is Rs. 3145. What length of this silk can be purchased for
Rs. 1445?
(a) 15 m (b) 16 m (c) 17 m (d) none of these

13. If 22.5 m of a uniform iron rod weighs 85.5 kg, what will be the length of 22.8 kg
of the same rod?
(a) 5 m (b) 6 m (c) 7 m (d) none of these

14. If the weight of 6 sheets of a paper is 162 grams, how many sheets of the same
quality of paper would weigh 13.5 kg?
(a) 300 (b) 400 (c) 500 (d) none of these

15. 1152 bars of soap can be packed in 8 cartoons of the same size. How many such
cartoons will be required to pack 3888 bars?
(a) 27 (b) 24 (c) 25 (d) none of these

16. If the thickness of a pile of 16 cardboards is 44mm, how many cardboards will be
there in a pile which is 71.5 cm thick?

Prepared by: M. S. KumarSwamy, TGT(Maths) Page - 43 -


(a) 270 (b) 260 (c) 250 (d) none of these
17. At a particular time of a day, a 7m high flagstaff casts a shadow which is 8.2m
long. What is the height of the building which casts a shadow 20.5 m in length at
the same time?
(a) 15.5 m (b) 16.5 m (c) 17.5 m (d) none of these

18. 15 men can build a 16.25m long wall up to a certain height in one day. How many
men should be employed to build a wall of the same height but of length 26m in
one day?
(a) 27 (b) 24 (c) 25 (d) none of these

19. In a hospital, the monthly consumption of milk of 60 patients is 1350 litres. How
many patients can be accommodated in the hospital if the monthly ration of milk
is raised 1710 litres, assuming that the quota per head remains the same?
(a) 75 (b) 76 (c) 77 (d) none of these

20. The extension in an elastic string varies directly as the weigh hung on it. If a
weight of 150 g produces an extension of 2.8 cm, what weight would produce an
extension of 19.6 cm?
(a) 1.5 kg (b) 1.05 kg (c) 15 kg (d) none of these

21. A car travels 432 km on 48 litres. How far would it travel on 20 litres of petrol?
(a) 160 km (b) 180 km (c) 200 km (d) none of these

22. If 40m of a cloth costs Rs. 1940, how many metres can be bought for Rs. 727.50?
(a) 15 m (b) 16 m (c) 17 m (d) none of these

23. A private taxi charges a fare of Rs. 260 for a journey of 200 km, how much would
it travel for Rs. 279.50?
(a) 200 km (b) 215 km (c) 200 km (d) none of these

24. Manoj types 540 words during half an hour, how many words would he type in 6
minutes?
(a) 105 (b) 106 (c) 108 (d) none of these

25. Rohit bought 12 registers for Rs. 156, find the cost of 7 such register.
(a) Rs. 90 (b) Rs. 91 (c) Rs. 92 (d) none of these

26. Pranshu takes 125 minutes in walking a distance of 100m. What distance would
he cover in 315 minutes?
(a) 250 m (b) 252 m (c) 254 m (d) none of these

27. If the cost of 93m of a certain kind of plastic sheet is Rs. 1395, then what would it
cost to bury 105m of such plastic sheet?
(a) Rs. 1500 (b) Rs. 1550 (c) Rs. 1575 (d) none of these

28. Ranjita types 1080 words in one hour. What is her gross words a minute rate?
(a) 15 (b) 16 (c) 18 (d) none of these

29. 68 boxes of a certain commodity require a shelf-length of 13.6 m. How many


boxes of the same commodity would occupy a shelf-length of 20.4m?
(a) 104 (b) 106 (c) 102 (d) none of these
Prepared by: M. S. KumarSwamy, TGT(Maths) Page - 44 -
30. A worker is paid Rs. 200 for 8 days work. If he works for 20 days, how much will
he get?
(a) Rs. 500 (b) Rs. 550 (c) Rs. 575 (d) none of these

31. If 52 men can do a piece of work in 35 days, in how many days 28 men will do it?
(a) 65 days (b) 75 days (c) 80 days (d) none of these

32. If 56 men can do a piece of work in 42 days, how many men will do it in 14 days?
(a) 165 (b) 166 (c) 168 (d) none of these

33. 120 men have food provision for 200 days. After 5 days, 30 men died due to an
cancer. How long will the remaining food last?
(a) 260 days (b) 275 days (c) 250 days (d) none of these

34. If x and y varies inversely as each other and x = 10 when y = 6. Find y when x =
15.
(a) 5 (b) 6 (c) 4 (d) none of these

35. Shreya cycles to her school at an average speed of 12km/hr. It takes her 20
minutes to reach the school. If she wants to reach her school in 15 minutes, what
should be her average speed?
(a) 15 km/hr (b) 16 km/hr (c) 18 km/hr (d) none of these

36. 1000 soldiers in a fort has enough food for 20 days. But some soldiers were
transferred to another fort and the food lasted for 25 days. How many soldiers
were transferred?
(a) 100 (b) 200 (c) 800 (d) none of these

37. If x and y varies inversely as each other and x = 8 when y = 32. Find y when x =
16.
(a) 64 (b) 16 (c) 4 (d) none of these

38. If x and y varies inversely as each other and x = 8 when y = 10. Find y when x =
2.
(a) 40 (b) 16 (c) 4 (d) none of these

39. If x and y varies inversely as each other and x = 2 when y = 40. Find x when x =
20.
(a) 40 (b) 16 (c) 4 (d) none of these

40. If a and b varies inversely as each other and a = 16 when b = 4. Find b when a = 8.
(a) 2 (b) 8 (c) 4 (d) none of these

41. If x and y varies directly as each other and x = 4 when y = 16. Find y when x = 9.
(a) 48 (b) 36 (c) 4 (d) none of these

42. If x and y varies directly as each other and x = 4 when y = 16. Find y when x = 12.
(a) 48 (b) 36 (c) 4 (d) none of these

43. If x and y varies directly as each other and x = 4 when y = 16. Find y when x = 1.
(a) 48 (b) 36 (c) 4 (d) none of these

Prepared by: M. S. KumarSwamy, TGT(Maths) Page - 45 -


44. If x and y varies directly as each other and x = 4 when y = 16. Find y when x = 3.
(a) 48 (b) 36 (c) 4 (d) none of these

45. If 36 men can do a piece of work in 25 days, in how many days will 15 men do it?
(a) 60 days (b) 75 days (c) 50 days (d) none of these

46. A work force of 50 men with a contractor an finish a piece of work in 5 months.
In how many months the same work can be completed by 125 men?
(a) 2 (b) 8 (c) 4 (d) none of these

47. 1200 men can finish a stock of food in 35 days. How many more men should join
the hostel so that the same stock may lst for 25 days?
(a) 480 (b) 360 (c) 400 (d) none of these

48. In a hostel of 50 girls, there are food provisions for 40 days. If 30 more girls join
the hostel, how long will these provisions last?
(a) 26 days (b) 20 days (c) 25 days (d) none of these

49. 18 men can reap a field in 35 days. For reaping the same field in 15 days, how
many men are required?
(a) 48 (b) 36 (c) 42 (d) none of these

50. 55 cows can graze a field in 16 days. How many cows will graze the same field in
10 days?
(a) 88 (b) 66 (c) 44 (d) none of these

TIME AND DISTANCE

IMPORTANT FACTS AND FORMULAE

Distance
Speed  ;
Time
Distance
Time  ;
Speed
Distance  Speed  Time
5
1. x km / hr  x  m/ s
18
18
2. x m/ s  x km / hr
5
4. If the ratio of the speeds of A and B is a:b , then the ratio of the times taken by
them to cover the same distance is 1 : 1 or b : a
a b
5. Suppose a man covers a certain distance at x km/ hr and an equal distance at y km /
hr . Then , the average speed during the whole journey is 2 xy km / hr
x y
Prepared by: M. S. KumarSwamy, TGT(Maths) Page - 46 -
MCQ QUESTIONS FOR PRACTICE

1. If a person walks at 14 km/hr instead of 10 km/hr, he would have walked 20 km


more. The actual distance travelled by him is:?
a)50 km b)56 km c)70 km d)80 km

2. The speed of a car increases by 2 kms after every one hour. If the distance
travelling in the first one hour was 35 kms. what was the total distance travelled in
12 hours??
a)456 kms b)482 kms c)552 kms d)556 kms

3. A person travels from P to Q at a speed of 40 kmph and returns by increasing his


speed by 50%. What is his average speed for the both the trips??
a)36 kmph b)40 kmph c)45 kmph d)48 kmph

4. Two trains 142 meter & 138 meter is length are running towards each other on
parallel line one at rate of 32 km/hr and another at the rate of 40 km/hr in what
time they will clear off each other.?
a)13 b)14 c)15 d)16

5. A train covers a distance in 50 min, if it runs at a speed of 48kmph on an average.


The speed at which the train must run to reduce the time of journey to 40min will
be.?
a)45 kmph b)60 kmph c)75 kmph d)80 kmph

6. A train can travel 50% faster than a car. Both start from point A at the same time
and reach point B 75 kms away from A at the same time. On the way, however,
the train lost about 12.5 minutes while stopping at the stations. The speed of the
car is:?
a)100 kmph b)110 kmph c)120 kmph d)130 kmph

7. A person croses a 600 m long street in 5 minutes. What is his speed in km per
hour??
a)3.6 b)7.2 c)8.4 d)10

8. A person has to cover a distance of 6 km in 45 minutes. If he covers one-half of


the distance in two-thirds of the total time, to cover the remaining distance in the
remaining time, his speed (in km/hr) must be:?
a)6 b)8 c)12 d)15

9. A person during his journey travels it in three parts, with speed of 10km/hr, 60
km/hr and 30 km/hr respectively. Find his average speed for the whole journey?
a)33.33 b)50 c)20 d)32.5

10. Sachin can cover a distance in 1hr 24min by covering 2/3 of the distance at 4
kmph and the rest at 5kmph.the total distance is??
a)5 km b)6 km c)7 km d)8 km

11. It takes eight hours for a 600 km journey, if 120 km is done by train and the rest
by car. It takes 20 minutes more, if 200 km is done by train and the rest by car.
The ratio of the speed of the train to that of the cars is: ?
a)2 : 3 b)3 : 2 c)3 : 4 d)4 : 3

Prepared by: M. S. KumarSwamy, TGT(Maths) Page - 47 -


12. A car moves at the speed of 80 km/hr. what is the speed of the car in metres per
second? ?
1 2 2
a)8 m/sec b)20 m/sec c)21 m/sec d)22 m/sec
9 9 9

13. A train starts from Delhi at 6:00 am and reaches Ambala cantt. at 10am. The other
train starts from Ambala cantt. at 8am and reached Delhi at 11:30 am, If the
distance between Delhi and Ambala cantt is 200 km, then at what time did the two
trains meet each other?
a)8:46 am b)8:30 am c)8:56 am d)8:50 am

14. A farmer travelled a distance of 61 km in 9 hours. He travelled partly on foot @ 4


km/hr and partly on bicycle @ 9 km/hr. The distance travelled on foot is: ?
a)14 km b)15 km c)16 km d)17 km

15. A and B walk a circulat track. They start at 8 a.m from the same point in the
opposite directions. A and B walk at a speed 0f 2 rounds per hour and 3 rounds
per hour respectively. How many times shall they cross each other before 9.30
a.m? ?
a)5 b)7 c)9 d)11

16. Two stations A and B are 110 km apart on a straight line. One train starts from A
at 7 am and travel towards B at 20 km/hr speed. Another train starts from B at 8
am and travel towards A at 25 km/hr speed. At what time will they meet??
a)9 am b)10 am c)11 am d)None of these

17. A man covered a certain distance at some speed. Had he moved 3 kmph faster, he
would have taken 40 minutes less. If he had moved 2 kmph slower, he would have
taken 40 minutes more. The distance (in km) is:?
a)35 b)37 c)39 d)40

18. The distance between two cities A and B is 330 Km. A train starts from A at 8
a.m. and travel towards B at 60 km/hr. Another train starts from B at 9 a.m and
travels towards A at 75 Km/hr. At what time do they meet? ?
a)10 a.m b)10.30 a.m c)11 a.m d)11.30 a.m

19. A man can row 4.5 km/hr in still water and he finds that it takes him twice as long
to row up as to row down the river. Find the rate of the stream. ?
a)2 km/hr b)2.5 km/hr c)1.5 km/hr d)1.75 km/hr

20. A man in a train notices that he can count 21 telephone posts in one minute. If
they are known to be 50 metres apart, then at what speed is the train travelling? ?
a)55 km/hr b)57 km/hr c)60 km/hr d)63 km/hr
21. Mac travels from A to B a distance of 250miles in 5½ hours. He returns to A in 4
hours 30 minutes. His average speed is:
(a) 42 mph (b) 49 mph (c) 48 mph (d) 50 mph

22. A boy goes to his school from his house at a speed of 3 km/hr and returns at a
speed of 2 km/hr. If he takes 5 hours in going and coming, the distance between

Prepared by: M. S. KumarSwamy, TGT(Maths) Page - 48 -


his house and school is:
(a) 8.5 km (b) 5.5 km (c) 6 km (d) 9 km

23. The average speed of a train in the onward journey is 25% more than that in the
return journey. The train halts for one hour on reaching the destination. The total
time taken for the complete to and for journey is 17 hours, covering a distance of
800 km. The speed of the train in the onward journey is:
(a) 50 km/hr (b) 53 km/hr (c) 52 km/hr (d) 56.25 km/hr

24. A man on tour travels first 160 km at 64 km/hr and the next 160 kin at 80 km/hr.
The average speed for the first 320 km of the tour is:
(a) 35.55 km/hr (b) 38 km/hr (c) 71.11 km/hr (d) 75 km/hr

25. A boy rides his bicycle 10 km at an average speed of 12 km/hr and again travels
12 km at an average speed of 10 km/hr. His average speed for the entire trip is
approximately :
(a) 10.4 km/hr (b) 10.8 km/hr (c) 12 km/hr (d) 14
km/hr

26. A car travels the first one-third of a certain distance with a speed of 10 km/hr, the
next one- third distance with a speed of 20 km/hr, and the last one-third distance
with a speed. of 60 km/hr. The average speed of the car for the whole journey is:
(a) 18 km/hr (b) 34 km/hr (c) 35 km/hr (d) 39 km/hr

27. A motorist covers a distance of 39 km in 45 minutes by moving at a speed of x


kmph for the first 15minutes, thenmoving at double the speed for the next 20
minutes and then again moving at his original speed for the rest of the journey
Then, x is equal to:
(a) 31.2 (b) 36 (c) 42 (d) 54

28. Mary jogs 9 km at a speed of 6 km per hour. At what speed would she need to jog
during the next 1.5 hours to have an average of 9 km per hour for the entire
jogging session?
(a) 9 kmph (b) 13 kmph (c) 12 kmph (d) 15 kmph

29. A car traveling with 5/7 of its actual speed covers 42 km in 1 hr 40min 48 sec.
Find the actual speed of the car.
(a) 17 km/hr (b) 32 km/hr (c) 31 km/hr (d) 35 km/hr

30. A train running at 7/11 of its own speed reached a place in 22 hours. How much
time could be saved if the train would have run at its own speed?
(a) 9 hours (b) 8 hours (c) 14 hours (d) 20 hours

31. A man can reach a certain place in 30 hours. If he reduces his speed by 1/15th, he
goes 10 km less in that time. Find his speed.
(a) 7 km/hr (b) 5 km/hr (c) 52 km/hr (d) 8 km/hr

32. Walking 6/7th of his usual speed, a man is 12 minutes too late. The usual time
taken by him to cover that distance is:
(a) 2 hour (b) 1 hr 12 min. (c) 1 hr 15 min. (d) 3 hr 20 min
33. A train when moves at an average speed of 40 kmph, reaches its destination on
time. When its average speed becomes 35 kmph, then it reaches its destination 15
Prepared by: M. S. KumarSwamy, TGT(Maths) Page - 49 -
minutes late. Find the length of journey.
(a) 50 km (b) 60 km (c) 70 km (d) 90 km

34. Robert is traveling on his cycle and has calculated to reach point A at 2 PM. if he
travels at 10 kmph; he will reach there at 12 noon if he travels at 15 kmph. At
what speed must he travel to reach A at 1 P.M.?
(a) 8 kmph (b) 13 kmph (c) 12 kmph (d) 16 kmph

35. If a train runs at 40 kmph, it reaches its destination late by 11minutes but if it runs
at 50 kmph, it is late by 5 minutes only. The correct time for the train to complete
its journey is:
(a) 13 min. (b) 17 min. (c) 19 min. (d) 22 min

36. A man covered a certain distance at some speed. Had he moved 3 kmph faster, he
would have taken 40 minutes less. If he had moved 2 kmph, slower, he would
have taken 40 minutes more. The distance (in km) is :
(a) 353 (b) 368 (c) 372 (d) 40

37. What is the time taken by a train running at 18km/h to cross a man standing on a
platform, the length of the train being 120 m?
(a) 22 s (b) 44 s (c) 12 s (d) 24 s

38. A man is walking at a speed of 10 km/h. After every kilometer, he takes rest for
5min.Howmuch time will he take to cover a distance of 5 km?
(a) 20 min (b) 30 min (c) 40 min (d) 50 min

39. Walking 5/7 of his usual rate, a boy reaches his school 6 min late. Find his usual
time to reach school.
(a) 10 min (b) 12 min (c) 15 min (d) 18 min

40. If I walk at 4 km/h, I miss the bus by 10 min. If I walk at 5 km/h, I reach 5 min
before the arrival of the bus. How far I walk to reach the busstand?
(a) 5 km (b) 5.5 km (c) 6 km (d) 7.5 km

SIMPLE AND COMPOUND INTEREST

IMPORTANT FACTS AND FORMULAE


1.. Principal: The money borrowed or lent out for a certain period is called the
principal or the sum.
2. Interest: Extra money paid for using other's money is called interest.
3. Simple Interest (S.I.) : If the interest on a sum borrowed for a certain period is
reckoned uniformly, then it is called simple interest.
Let Principal = P, Rate = R% per annum (p.a.) and Time = T years. Then,
P  R T
(i) S .I . 
100
100  I 100  I 100  I
(ii) P  ; R ; T
R T P T PR
Compound Interest: Sometimes it so happens that the borrower and the lender agree
to fix up a certain unit of time, say yearly or half-yearly or quarterly to settle the
Prepared by: M. S. KumarSwamy, TGT(Maths) Page - 50 -
previous account.

In such cases, the amount after first unit of time becomes the principal for the
second unit,the amount after second unit becomes the principal for the third unit and
so on.

After a specified period, the difference between the amount and the money
borrowed is called the Compound Interest (abbreviated as C.I.) for that period.

Let Principal = P, Rate = R% per annum, Time = n years.

I. When interest is compound Annually:


n
 R 
Amount = P 1  
 100 
II. When interest is compounded Half-yearly:
2n
 R 
Amount = P 1  
 200 

III. When interest is compounded Quarterly:


4n
 R 
Amount = P 1  
 400 

IV. When interest is compounded Annually but time is in fraction, say


2
3 years.
5
3
 R   2R 
Amount = P 1    1  
 100   500 
V. When Rates are different for different years, say Rl%, R2%, R3% for 1st,
2nd and 3rd year respectively.
 R  R  R 
Then, Amount = P 1  1  1  2  1  3 
 100  100  100 

VI. Present worth of Rs.x due n years hence is given by :


x
Present Worth = n
 R 
 1  
 100 

MCQ QUESTIONS FOR PRACTICE

1. The simple interest on Rs. 5000 for 219 days at 4% per annum is
a) Rs 126.50 b) Rs 120 c) Rs 125 d) Rs. 43.80

2. Find the simple interest on Rs. 2500 for 2 years 6 months at 6% per annum.
a) Rs 350 b) Rs 375 c) Rs 750 d) none of these

3. What sum will amount to Rs. 4590 at 12% per annum simple interest in 3 years?
a) Rs 3500 b) Rs 3375 c) Rs 3750 d) none of these

Prepared by: M. S. KumarSwamy, TGT(Maths) Page - 51 -


4. In what time will Rs. 1860 amount to Rs. 2278.50, if simple interest is calculated
at 9% per annum?
1
a) 2 years b) 2 years c) 3 years d) 4 years
2

5. At what rate percent per annum will Rs. 1650 amount to Rs. 2046 in 3 years?
2
a) 8% b) 4% c) 6% d) 16 %
3

16
6. Simple interest on a certain sum is of the sum. Find the rate percent and the
25
time if both are numerically equal.
1
a) 6 years b) 2 years c) 8 years d) 4 years
2

7. At what rate percent per annum simple interest will a sum treble itself in 16 years?
2
a) 12% b) 12.5% c) 15% d) 16 %
3

8. A sum of money at simple interest amounts to Rs. 696 in 2 years and Rs. 840 in 5
years. The sum is
a) Rs 500 b) Rs 600 c) Rs 560 d) Rs. 620

9. In what time will Rs. 1600 amount to Rs. 1768 at 6% per annum simple interest?
1 1 3 1
a) 1 years b) 2 years c) 1 years d) 1 years
4 2 4 2

10. A sum amounts to Rs. 3720 in 8 months at 5% per annum simple interest. The
sum is
a) Rs 3500 b) Rs 3600 c) Rs 3560 d) Rs. 3620

11. At what rate percent per annum simple interest will a sum be double itself in 8
years?
1
a) 15% b) 14% c) 16% d) 12 %
2

12. At simple interest a sum becomes of itself in 5 years. The rate of interest percent
per annum is
a) 8% b) 5% c) 10% d) 12%

13. What amount Heena has to pay at the end of 2 years on a sum of Rs 20,000 at an
interest of 8% compounded annually?
a) Rs 23328 b) Rs 23000 c) Rs 23350 d) Rs
23450

14. What will be the bill amount if the cost of a toy is Rs 450 and the sales tax charge
is 5%?
a) Rs 472.5 b) Rs 437.5 c) Rs 460 d) Rs 455
Prepared by: M. S. KumarSwamy, TGT(Maths) Page - 52 -
15. The compounded interest on Rs. 50,000 at 8% per annum for 2 years,
compounded annually is
a) Rs 8000 b) Rs 8250 c) Rs 8350 d) Rs 8640

16. At what rate percent per annum will a sum of Rs. 7500 amount to Rs. 8427 in 2
years compounded annually?
a) 5% b) 4% c) 6% d) 8%

17. The compounded interest on Rs. 30000 at 7% per annum for a certain time is Rs.
4347. The time is
1
a) 2 years b) 2 years c) 3 years d) 4 years
2
1
18. The principal that amounts to Rs. 4913 in 3 years at 6 % per annum compound
4
interest, compounded annually is
a) Rs 3096 b) Rs 4076 c) Rs 4085 d) Rs 4096

19. If the simple interest on a sum of money at 5% per annum for 3 years is Rs. 1200,
then the compound interest on the same sum for he same period at the same rate
will be
a) Rs 1225 b) Rs 1236 c) Rs 1248 d) Rs 1261
1
20. If the compound interest on a sum for 2 years at 12 % per annum is Rs. 510, the
2
simple interest on the same sum at the same rate for same period of time is
a) Rs 400 b) Rs 450 c) Rs 460 d) Rs 480
21. The difference between compound interest and simple interest on a sum of Rs.
15000 for 2 years is Rs. 96. What is the rate of interest percent per annum?
a) 10% b) 12% c) 6% d) 8%
22. The compounded interest on Rs. 8,000 at 5% per annum for 3 years, compounded
annually is
a) Rs 1225 b) Rs 1236 c) Rs 1248 d) Rs 1261
1
23. The compounded interest on Rs. 6400 at 7 % per annum for 2 years,
2
compounded annually is
a) Rs 7096 b) Rs 7396 c) Rs 7000 d) none of these

24. In what time will be Rs. 1000 amount to Rs. 1331 at 10% per annum compounded
annually?
1
a) 2 years b) 2 years c) 3 years d) 4 years
2
25. The yearly interest payable on a deposit of Rs. 250 at 5.5% p.a. simple interest is:
a) Rs. 137.50 b) Rs. 13.75 c)Rs. 12.50 d)Rs. 125.00 e)Rs. 17.35

26. The interest on Rs. 12 167 invested for 5 years at 2.5% simple interest p.a. would
be nearest to:
a)Rs. 1220.87 b)Rs. 1521 c)Rs. 1520.88 d)Rs. 152.10 e)Rs. 152.09

Prepared by: M. S. KumarSwamy, TGT(Maths) Page - 53 -


27. Sarah received Rs. 75 interest on her term deposit of Rs. 450 over 3 years. The
simple interest rate per annum was:
a)56% b)5.6% c)22% d)0.55% e)11.2%

28. Tom earned Rs. 504 when he invested Rs. 16 820 for 8 months. His rate of simple
interest was:
a)0.37% p.a. b)2.00% p.a. c)3.00% p.a. d)4.49% p.a. e)5.00% p.a.

29. John invested a sum of money, which earned Rs. 750 simple interest at the rate of
6% per annum over 2 years. The amount invested was:
a)Rs. 88.80 b)Rs. 6250 c)Rs. 3125 d)Rs. 2125 e)None of these.
30. Cathy invested Rs. 8200 at the rate of 4.5% p.a. It earned Rs. 738 simple interest.
The period of investment was:
a)6 months b)1 year c)2 years d)2 years 6 months e)3 years

31. A 500-g packet of chocolate costs Rs. 2.50. Assuming inflation averages 2.8% per
annum over each of the next 3 years, how much will the chocolate cost in three
years?
a)Rs. 2.64 b)Rs. 2.72 c)Rs. 2.79 d)Rs. 2.87 e)Rs. 2.95

32. An investment of Rs. 10 000 at the rate of 8% per annum, compounded quarterly,
will reach Rs. 14 800 in close to:
a)1 year b)2 years c)3 years d)4 years e)5 years

33. An investment of Rs. 10 000 to be invested for a period of 5 years and


compounded quarterly at 3.75% p.a. will have a future value closest to which of
the following:
a)Rs. 10 480 b)Rs. 12 020 c)Rs. 12 050 d)Rs. 20 900 e)None of the
above

34. Tom has the choice of investing his money in compound interest with rests as
indicated below. Which situation would return him the most interest?
a)yearly rests b)six-monthly rests c)quarterly rests d)monthly rests e)daily rests

35. The growth of an investment over a period of time is shown as a graph which
depicts a straight line. Which of the following investments would depict a straight
line?
a)6% p.a. simple interest b)6% p.a. compounding yearly
c)6% p.a. compounding quarterly d)6% p.a. compounding monthly
e)6% p.a. compounding daily

36. How long would it take for Rs. 5000 invested at 5% p.a. compound interest with
yearly rests to double in value?
a)5 years b)7 years c)10 years d)14 years e)20 years

37. An interest rate of 4.5% p.a. compounding monthly is equivalent to an effective


interest rate of:
a)4.50% p.a. b)4.55% p.a. c)4.57% p.a. d)4.59% p.a. e)4.60% p.a.

Prepared by: M. S. KumarSwamy, TGT(Maths) Page - 54 -


38. Which of the following compounding rates is equivalent to an effective interest
rate of 2.75% p.a.?
a)2.7% p.a. compounding six-monthly b)2.75% p.a. compounding yearly
c)2.6% p.a. compounding quarterly d)2.6% p.a. compounding
monthly
e)2.6% p.a. compounding daily

39. Two banks pay simple interest on short-term deposits. Bank A pays 5% p.a. over
3 years, and Bank B pays 5.5% p.a. for 2 12 years. The difference between the two
banks’ final payout figure, if Rs. 10 000 was invested in each account is:
a)Rs. 0 b)Rs. 125 c)Rs. 250 d)Rs. 1375 e)Rs. 1500

40. Paul wishes to invest Rs. 10 000 for a period of 5 years. Which of the following
investments would be best for him?
a)6.7% p.a. simple interest
b)6.75% p.a. compound interest with yearly rests
c)6.5% p.a. compound interest with quarterly rests
d)6.25% p.a. compound interest with monthly rests
e)6% compound interest with daily rests.

41. An investment of Rs. 6000 was placed for 3 years at 4.25% p.a. compounded
annually. How much more would be collected if the investment was compounded
quarterly?
a)No more b)Rs. 13.36 c)Rs. 32.97 d)Rs. 46.33 e)Rs. 52.29

42. An automobile financier claims to be lending money at simple interest, but he


includes the interest every six months for calculating the principal. If he is
charging an interest of 10%, the effective rate of interest becomes:
a)13% b)10.25% c)15% d)11% e)None of these

43. A sum of money at simple interest amounts to Rs. 815 in 3 years and to Rs. 854
in 4 years. The sum is:
a)Rs. 850 b)Rs. 790 c)Rs. 698 d)Rs. 800 e)Rs. 600

44. Sum of money becomes Rs. 13,380 after 3 years and Rs. 20,070 after 6 years on
compound interest. The sum is:
a)Rs. 9200 b)Rs. 9000 c)Rs. 8920 d)Rs. 9040 e)Rs. 9500

45. A sum of Rs. 12,000 deposited at compound interest becomes double after 5
years. After 20 years, it will become:
a)Rs. 1,10,000 b)Rs. 1,30,000 c)Rs. 1,24,000 d)Rs. 1,92,000 e)Rs. 1,50,000

46. A sum of money placed at compound interest doubles itself in 5 years. It will
amount to eight times itself at the same rate of interest in:
a)7 years b)12 years c)15 years d)30 years e)21 years

47. If a sum on compound interest becomes three times in 4 years, then with the same
interest rate, the sum will become 27 times in:
a)11 years b)12 years c)24 years d)38 years e)21 years

Prepared by: M. S. KumarSwamy, TGT(Maths) Page - 55 -


48. The least number of complete years in which a sum of money put out at 20%
compound interest will be more than doubled is:
a)7 b)4 c) 5 d) 8 e) 7

49. A man borrows Rs. 2550 to be paid back with compound interest at the rate of
4% per annum by the end of 2 years in two equal yearly installments. How much
will each installment be?
a)Rs.1275 b)Rs.1383 c) Rs.1352 d) Rs.1287 e) Rs.1250

50. What annual payment will discharge a debt of Rs. 1025 due in 2 years at the rate
of 5% compound interest?
a)Rs.650 b) Rs.551.25 c) Rs.560 d) Rs.660.75 e) Rs.600

51. The difference between compound interest and simple interest on an amount of
Rs. 15,000 for 2 years is Rs. 96. What is the rate of interest per annum?
(a) 8 (b) 11 (c) 12 (d) None of these

52. The difference between simple and compound interests and compounded annually
on a certain sum of money for 2 years at 4% per annum is Re. 1. The sum (in Rs.)
is:
(a) 625 (b) 620 (c) 640 (d) 660

53. The compound interest on a sum of money for 2 years is Rs. 832 and the simple
interest on the same sum for the same period is Rs. 800. The difference between
the compound interest and the simple interest for 3 years will be:
(a) Rs. 50 (b) Rs. 67 (c) Rs. 98.56 (d) Rs. 75.45

54. The difference between the simple interest on a certain sum at the rate of 10% per
annum for 2 years and compound interest which is compounded every 6 months is
Rs. 124.05. What is the principal sum?
(a) Rs. 9000 (b) Rs. 8000 (c) Rs. 10,000 (d) Rs. 13,000

55. The difference between the simple interest on a certain sum at the rate of 10% per
annum for 2 years and compound interest which is compounded every 6 months is
Rs. 124.05. What is the principal sum?
(a) Rs. 9000 (b) Rs. 8000 (c) Rs. 10,000 (d) Rs. 13,000

56. A sum of money lent at compound interest for 2 years at 20% per annum would
fetch Rs. 482 more, if the interest was payable half-yearly than if it was payable
annually. The sum is:
(a) Rs. 12,000 (b) Rs. 20,000 (c) Rs. 40,000 (d) Rs. 60,000

57. On a sum of money, the simple interest for 2 years is Rs. 660, while the
compound interest is Rs. 696.30, the rate of interest being the same in both the
cases. The rate of interest is:
(a) 13% (b) 14% (c) 12% (d) 11%

58. Sum of money becomes Rs. 13,380 after 3 years and Rs. 20,070 after 6 years on
compound interest. The sumis:
(a) Rs. 9200 (b) Rs. 9000 (c) Rs. 8920 (d) Rs. 9040

Prepared by: M. S. KumarSwamy, TGT(Maths) Page - 56 -


59. A sum of Rs. 12,000 deposited at compound interest becomes double after 5
years. After 20 years, it will become:
(a) Rs. 1,10,000 (b) Rs. 1,30,000 (c) Rs. 1,24,000 (d) Rs. 1,92,000

60. A sum of money placed at compound interest doubles itself in 5 years. It will
amount to eight times itself at the same rate of interest in:
(a) 7 years (b) 12 years (c) 15 years (d) 30 years

MENSURATION

IMPORTANT CONCEPTS OF AREA

I.RESULTS ON TRIANGLES:
1.Sum of the angles of a triangle is 180 degrees.
2.Sum of any two sides of a triangle is greater than the third side.
3.Pythagoras theorem:
In a right angle triangle, (Hypotenuse)2 = (base)2 + (Height)2
4.The line joining the midpoint of a side of a triangle to the opposite vertex is called
the
MEDIAN
5.The point where the three medians of a triangle meet is called CENTROID.
Centroid divides each of the medians in the ratio 2:1.
6.In an isosceles triangle, the altitude from the vertex bi-sects the base
7.The median of a triangle divides it into two triangles of the same area.
8.Area of a triangle formed by joining the midpoints of the sides of a given triangle is
one-fourth of the area of the given triangle.

II.RESULTS ON QUADRILATERALS:
1. The diagonals of a parallelogram bisects each other .
2. Each diagonal of a parallelogram divides it into two triangles of the same area
3. The diagonals of a rectangle are equal and bisect each other.
4. The diagonals of a square are equal and bisect each other at right angles.
5. The diagonals of a rhombus are unequal and bisect each other at right angles.
6. A parallelogram and a rectangle on the same base and between the same
parallels are equal in area.
7. Of all the parallelograms of a given sides, the parallelogram which is a
rectangle has the greatest area.
IMPORTANT FORMULAE
I.1.Area of a rectangle=(length x breadth)
Therefore length = (area/breadth) and breadth=(area/length)
2.Perimeter of a rectangle = 2 x (length + breadth)
1
II.Area of a square = (side)2 = (diagonal)2
2
III Area of four walls of a room = 2x(length + breadth)x(height)
1
IV 1.Area of the triangle= (base x height)
2
2. Area of a triangle = s ( s  a)( s  b)( s  c) , where a, b, c are the sides of a

Prepared by: M. S. KumarSwamy, TGT(Maths) Page - 57 -


abc
triangle and s =
2
3
3.Area of the equilateral triangle = ( side) 2
4
a
4.Radius of incircle of an equilateral triangle of side a = 3
2
a
5.Radius of circumcircle of an equilateral triangle of side a =
3

V.1.Area of the parellogram =(base x height)


1
2.Area of the rhombus = (product of the diagonals)
2
1
3.Area of the trapezium = (size of parallel sides) x distance between them
2
2
VI 1.Area of a circle =  r ,where r is the radius
2. Circumference of a circle = 2 r .

3. Length of an arc =  2 r where θ is the central angle
360
 1
4. Area of a sector =   r 2 = (arc  r )
360 2
2
r
VII. 1. Area of a semi-circle =
2
2. Circumference of a semi-circle =  r .

IMPORTANT FORMULAE ON VOLUME AND SURFACE AREAS

I. CUBOID
Let length = 1, breadth = b and height = h units. Then,
1. Volume = (1 x b x h) cubic units.
2. Surface area= 2(lb + bh + lh) sq.units.
3. Diagonal.= l 2  b 2  h 2 units

II. CUBE
Let each edge of a cube be of length a. Then,
1. Volume = a3 cubic units.
2. Surface area = 6a2 sq. units.
3. Diagonal = 3a units.

III. CYLINDER
Let radius of base = r and Height (or length) = h. Then,
1. Volume =  r 2 h cubic units.
2. Curved surface area = 2 rh units.
3. Total surface area = 2 r (r  h) sq. units

IV. CONE
Let radius of base = r and Height = h. Then,
1. Slant height, l  r 2  h 2

Prepared by: M. S. KumarSwamy, TGT(Maths) Page - 58 -


1 2
2. Volume =  r h cubic units.
3
3. Curved surface area =  rl sq. units.
4. Total surface area =  r (r  l ) sq. units.

V. SPHERE
Let the radius of the sphere be r. Then,
4
1. Volume =  r 3 cubic units.
3
2. Surface area = 4 r 2 sq. units.

VI. HEMISPHERE
Let the radius of a hemisphere be r. Then,
2
1. Volume =  r 3 cubic units.
3
2. Curved surface area = 2 r 2 sq. units.
3. Total surface area = 3 r 2 units.
Remember: 1 litre = 1000 cm3 and 1m3 = 1000 litres

1. The area of a rectangular sheet is 500 cm2. If the length of the sheet is 25 cm, what
is its width?
(a) 20 cm (b) 17 cm (c) 30 cm (d) 25 cm

2. If the area of rectangle increases from 2 cm2 to 4 cm2 the perimeter will
(a) increase (b) decrease (c) remains same (d) none of these

3. Which figure encloses more area : a square of side 2 cm ; a rectangle of side 3


cm & 2 cm ;An equilateral triangle of side 4 cm
(a) rectangle (b) square (c) triangle (d) same of rectangle & square

4. ABC is isosceles in which AE  BC, AE = 6 cm , BC = 9 cm, the area of


ABC is
(a) 27 cm2 (b) 54 cm2 (c) 22.5 cm2 (d) 45 cm2

5. Which of the following has the formula : Base x Height


(a) area of parallelogram (b) area of quadrilateral
(c) area of triangle (d) area of trapezium

6. If the area of the triangle is 36 cm2 and the height is 3 cm, the base of the triangle
will be
(a) 12 cm (b) 39 cm (c) 108 cm (d) 24 cm

7. The height of parallelogram whose area is 35 cm2 and altitude 7 cm


(a) 5 cm (b) 5 cm2 (c) 245 cm (d) 245 cm2

8. Area of triangle whose base is 15 cm and corresponding altitude is 6 cm will be


(a) 45 cm2 (b) 90 cm2 (c) 45 cm (d) 90 cm

9. Find the area of a right triangle whose base is 3 cm, perpendicular is 2 cm and
hypotenuse is 5 cm.
(a) 3 cm2 (b) 7.5 cm2 (c) 5 cm2 (d) 6 cm
Prepared by: M. S. KumarSwamy, TGT(Maths) Page - 59 -
10. What will be the area of circular button of radius 7 cm
(a) 154 cm2 (b) 49 cm2 (c) 154 cm (d) 3.14 x 7 cm2

11. The circumference of circle whose diameter is 14 cm will be


(a) 44 cm (b) 88 cm (c) 44 cm2 (d) 88 cm2
12. If the area of circle is 44 cm2 , the area of shaded portion will be
(a) 11 cm2 (b) 11 cm
(c) 22 cm2 (d) 22 cm2

13. If the radius of pipe is 1 cm , the circumference of pipe will be


(a) 62.8 cm (b) 6.28 cm (c) 62.8 cm2 (d) 6.28
cm

14. One hectare is equal to


(a) 100 m2 (b) 1000 m2 (c) 10,000 m2 (d) 10,000 m

15. The circumference of a circle with radius 7 cm is


(a) 11 cm (b) 22 cm (c) 44 cm (d) 49 cm

16. The area of a circle is 49 cm2. Its circumference is


(a) 7 cm (b) 14 cm (c) 21 cm (d) 28 cm

17. The perimeter of circular field is 242cm. The area of the field is
(a) 9317 cm2 (b) 18634 cm2 (c) 4658.5 cm2 (d) none of these

18. The area of a circle is 38.5 cm2. Its circumference is


(a) 62 cm (b) 12.1 cm (c) 11 cm (d) 22 cm

19. The difference between the circumference and radius of a circle is 37 cm. The area
of the circle is
(a) 111 cm2 (b) 184 cm2 (c) 154 cm2 (d) 259 cm2

20. The circumference of two circles are in the ratio 2 : 3. The ratio of their areas is
(a) 2 : 3 (b) 4 : 9 (c) 9 : 4 (d) none of these

21. The area of the square is the same as the area of the circle. Their perimeter re in
the ratio
(a) 1 : 1 (b)  : 2 (c) 2 :  (d) none of these

22. The areas of the two circle are in the ratio 4 : 9. The ratio of their circumference is
(a) 2 : 3 (b) 4 : 9 (c) 9 : 4 (d) 4 : 9

23. In making 1000 revolutions, a wheel covers 88 km. The diameter of the wheel is
(a) 14 m (b) 24 m (c) 28 m (d) 40 m

24. The diameter of a wheel is 40 cm. How many revolutions will it make an covering
176 m?
(a) 140 (b) 150 (c) 160 (d) 166

25. The radius of wheel is 0.25 m. How many revolutions will it make in covering 11
km?
Prepared by: M. S. KumarSwamy, TGT(Maths) Page - 60 -
(a) 2800 (b) 4000 (c) 5500 (d) 7000

26. Find the area of a circle whose circumference is 52.8 cm.


(a) 221.76 cm2 (b) 220.76 cm2 (c) 200.76 cm2 (d) none of these.

27. A steel wire when bent in the form of a square, encloses an area of 121 sq. cm.
The same wire is bent in the form of a circle. Find the area of the circle.
(a) 111 cm2 (b) 184 cm2 (c) 154 cm2 (d) 259 cm2

28. If the perimeter of a semicircular protractor is 36 cm, find the diameter.


(a) 14 cm (b) 16 cm (c) 18 cm (d) 12 cm

29. A bicycle wheel makes 5000 revolutions in moving 11 km. Find the diameter of
the wheel.
(a) 60 cm (b) 70 cm (c) 66 cm (d) 68 cm

30. The diameter of the wheels of a bus is 140 cm. How many revolutions per minute
must a wheel make in order to move a t a speed of 66km/hr?
(a) 240 (b) 250 (c) 260 (d) 270

31. The perimeter of regular polygon is


(a) no. of sides x lengths of one side (b) no. of sides + lengths of one side
(c) no. of sides – lengths of one side (d) no. of sides  lengths of one side

32. A wire is in the shape of a square of side 10 cm. If the wire is rebent into a
rectangle of length 12 cm, find its breadth.
(a) 12 cm (b) 7 cm (c) 8 cm (d) 9 cm

33. A paper is in the form of a rectangle ABCD in which AB = 18cm and BC = 14cm.
A semicircular portion with BC as diameter is cut off. Find the area of the
remaining paper (see in below figure).
(a) 175 cm2 (b) 165 cm2 (c) 145 cm2 (d) none of these

34. Find the area of the shaded region in the above sided figure. Take  = 3.14
(a) 75 cm2 (b) 72 cm2 (c) 70 cm2 (d) none of these

35. The area of a square and a rectangle are equal. If the side of the square is 40 cm
and the breadth of the rectangle is 25 cm, find the length of the rectangle.
(a) 60 cm (b) 62 cm (c) 64 cm (d) 68 cm

36. The perimeter of floor of rectangular hall is 250m. The cost of the white washing
its four walls is Rs. 15000. The height of the room is
(a) 5m (b) 4m (c) 6m (d) 8m

Prepared by: M. S. KumarSwamy, TGT(Maths) Page - 61 -


37. The perimeter of parallelogram PQRS is:
(a) 12 cm (b) 7 cm (c) 38 cm (d) 19 cm
P
12 cm Q

7 cm

R S

38. The length, breadth and height of a room is 5m, 4m and 3m. The cost of white
washing its four walls at the rate of Rs. 7.50 per m2 is
(a) Rs. 110 (b) Rs. 109 (c) Rs. 220 (d) Rs. 105

39. The breadth of a room is twice its height and is half of its length. The volume of
room is 512dm3. Its dimensions are
(a) 16 dm, 8 dm, 4 dm (b) 12 dm, 8 dm, 2 dm
(c) 8 dm, 4 dm, 2 dm (d) 10 dm, 15 dm, 20 dm

40. The area of three adjacent faces of a cube is x, y and z. Its volume V is
(a) V = xyz (b) V3 = xyz (c) V2 = xyz (d) none of these

41. Two cubes each of edge 12 cm are joined. The surface area of new cuboid is
(a) 140 cm2 (b) 1440 cm2 (c) 144 cm2 (d) 72 cm2
2
42. The curved surface area of a cylinder of height 14 cm is 88 cm . The diameter of
its circular base is
(a) 5cm (b) 4cm (c) 3cm (d) 2cm

43. It is required to make a closed cylindrical tank of height 1 m and base diameter
140cm from a metal sheet. How many square meters a sheet are required for the
same?
(a) 6.45m2 (b) 6.48m2 (c) 7.48m2 (d) 5.48m2.

44. A metal pipe is 77 cm long. Inner diameter of cross section is 4 cm and outer
diameter is 4.4 cm. Its inner curved surface area is:
(a) 864 cm2 (b) 968 cm2 (c) 768 cm2 (d) none of these

45. The diameter of a roller is 84 cm and its length is 120 cm. It takes 500 complete
revolutions to move once over to level a playground. The area of the playground
in m2 is:
(a) 1584 (b) 1284 (c) 1384 (d) 1184

46. A cylindrical pillar is 50 cm in diameter and 3.5 m in height. The cost of painting
its curved surface at the rate of Rs. 12.50 per m2 is:
(a) Rs. 68.75 (b) Rs. 58.75 (c) Rs. 48.75 (d) Rs. 38.75

47. The inner diameter of circular well is 3.5m. It is 10m deep. Its inner curved
surface area in m2 is:
(a) 120 (b) 110 (c) 130 (d) 140

Prepared by: M. S. KumarSwamy, TGT(Maths) Page - 62 -


48. In a hot water heating system there is a cylindrical pipe of length 28 m and
diameter 5 cm. The total radiating surface area in the system in m2 is:
(a) 6.6 (b) 5.5 (c) 4.4 (d) 3.4

49. The curved surface area of a right circular cone of slant height 10 cm and base
radius 7 cm is
(a) 120 cm2 (b) 220 cm2 (c) 240 cm2 (d) 140 cm2

50. The height of a cone is 16 cm and base radius is 12 cm. Its slant height is
(a) 10 cm (b) 15 cm (c) 20 cm (d) 8 cm

51. The curved surface area of a right circular cone of height 16 cm and base radius
12 cm is
(a) 753.6 cm2 (b) 1205.76 cm2 (c) 863.8 cm2 (d) 907.6 cm2

52. The curved surface area of a right circular cone of slant height 10 cm and base
radius 10.5 cm is
(a) 185 cm2 (b) 160 cm2 (c) 165 cm2 (d) 195 cm2

53. The slant height of a cone is 26 cm and base diameter is 20 cm. Its height is
(a) 24 cm (b) 25 cm (c) 23 cm (d) 35 cm
2
54. The curved surface area of a cone is 308 cm and its slant height is 14 cm. The
radius of its base is
(a) 8 cm (b) 7 cm (c) 9 cm (d) 12 cm

55. A conical tent is 10 m high and the radius of its base is 24 m. The slant height of
tent is
(a) 26 m (b) 28 m (c) 25 m (d) 27 m

56. The slant height and base diameter of a conical tomb are 25 m and 14 m
respectively. The cost of white washing its curved surface at the rate of Rs. 210
per 100 m2 is
(a) Rs. 1233 (b) Rs. 1155 (c) Rs. 1388 (d) Rs. 1432

57. A joker’s cap is in the form of cone of base radius 7 cm and height 24 cm. The
area of sheet to make 10 such caps is
(a) 5500 cm2 (b) 6500 cm2 (c) 8500 cm2 (d) 3500 cm2

58. A solid right cylinder cone is cut into two parts at the middle of its height by a
plane parallel to its base. The ratio of the volume of the smaller cone to the whole
cone is
(a) 1 : 2 (b) 1 : 4 (c) 1 : 6 (d) 1 : 8

59. The curved surface area of a sphere of radius 7 cm is:


(a) 516 cm2 (b) 616 cm2 (c) 716 cm2 (d) 880 cm2

60. The curved surface area of a hemisphere of radius 21 cm is:


(a) 2772 cm2 (b) 2564 cm2 (c) 3772 cm2 (d) 4772 cm2

61. The curved surface area of a sphere of radius 14 cm is:


(a) 2464 cm2 (b) 2428 cm2 (c) 2464 cm2 (d) none of these.

Prepared by: M. S. KumarSwamy, TGT(Maths) Page - 63 -


62. The curved surface area of a sphere of diameter 14 cm is:
(a) 516 cm2 (b) 616 cm2 (c) 716 cm2 (d) 880 cm2

63. Total surface area of hemisphere of radius 10 cm is


(a) 942 cm2 (b) 940 cm2 (c) 842 cm2 (d) 840 cm2

64. The radius of a spherical balloon increases from 7 cm to 14 cm s air is being


pumped into it. The ratio of surface area of the balloon in the two cases is:
(a) 4 : 1 (b) 1 : 4 (c) 3 : 1 (d) 1 : 3
65. A matchbox measures 4 cm x 2.5 cm x 1.5 cm. The volume of packet containing
12 such boxes is:
(a) 160 cm3 (b) 180 cm3 (c) 160 cm2 (d) 180 cm2
66. A cuboidal water tank is 6 m long, 5 m wide and 4.5 m deep. How many litre of
water can it hold?
(a) 1350 liters (b) 13500 liters (c) 135000 liters (d) 135 liters
67. A cuboidal vessel is 10 m long and 8 m wide. How high must it be made to hold
380 cubic metres of a liquid?
(a) 4.75 m (b) 7.85 m (c) 4.75 cm (d) none of these
68. The capacity of a cuboidal tank is 50000 litres. The length and depth are
respectively 2.5 m and 10 m. Its breadth is
(a) 4 m (b) 3 m (c) 2 m (d) 5 m
69. A godown measures 40 m × 25 m × 10 m. Find the maximum number of wooden
crates each measuring 1.5 m × 1.25 m × 0.5 m that can be stored in the godown.
(a) 18000 (b) 16000 (c) 15000 (d) 14000

70. A river 3 m deep and 40 m wide is flowing at the rate of 2 km per hour. How
much water will fall into the sea in a minute?
(a) 4000 m3 (b) 40 m3 (c) 400 m3 (d) 40000 m3

71. The circumference of the base of a cylindrical vessel is 132 cm and its height is 25
cm. How many litres of water can it hold?
(a) 33.75 litre (b) 34.65 litre (c) 35.75 litre (d) 38.75 litre

72. If the lateral surface of a cylinder is 94.2 cm2 and its height is 5 cm, then find
radius of its base
(a) 5cm (b) 4cm (c) 3cm (d) 6cm

73. It costs Rs 2200 to paint the inner curved surface of a cylindrical vessel 10 m
deep. If the cost of painting is at the rate of Rs 20 per m2, find radius of the base,
(a) 1.75 m (b) 1.85 m (c) 1.95 m (d) 1.65 m

74. The height and the slant height of a cone are 21 cm and 28 cm respectively. Find
the volume of the cone.
(a) 5546 cm3 (b) 7546 cm3 (c) 5564 m3 (d) 8546 cm3

75. Find the volume of the right circular cone with radius 6 cm, height 7 cm
(a) 254 cm3 (b) 264 cm3 (c) 274 cm2 (d) 284 cm3

76. The radius and height of a conical vessel are 7 cm and 25 cm respectively. Its
capacity in litres is
Prepared by: M. S. KumarSwamy, TGT(Maths) Page - 64 -
(a) 1.232 litre (b) 1.5 litre (c) 1.35 litre (d) 1.6 litre

77. The height of a cone is 15 cm. If its volume is 1570 cm3, find the radius of the
base.
(a) 12 cm (b) 10 cm (c) 15 cm (d) 18 cm
78. If the volume of a right circular cone of height 9 cm is 48 cm3, find the diameter
of its base.
(a) 12 cm (b) 10 cm (c) 6 cm (d) 8 cm

79. A conical pit of top diameter 3.5 m is 12 m deep. What is its capacity in kilolitres?
(a) 38.5 kl (b) 48.5 kl (c) 39.5 kl (d) 47.5 kl
80. Find the capacity in litres of a conical vessel with radius 7 cm, slant height 25 cm
(a) 1.232 litre (b) 1.5 litre (c) 1.35 litre (d) none of these
81. The diameter of the moon is approximately one-fourth of the diameter of the
earth. What fraction of the volume of the earth is the volume of the moon?
1 1 1 1
(a) (b) (c) (d)
64 32 16 48
82. The dimensions of a cuboid are 50 cm x 40 cm x 10 cm. Its volume in litres is:
(a) 10 litres (b) 12 litres (c) 20 litres (d) 25 litres
83. The volume of a cuboidal tank is 250 m3. If its base area is 50 m2 then depth of the
tank is
(a) 5 m (b) 200 m (c) 300 m (d) 12500 m

84. The volume of a cuboidal solid of length 8 m and breadth 5 m is 200 m3. Find its
height.
(a) 5 m (b) 6 m (c) 15 m (d) 18 m

85. The curved surface area of a sphere is 616 cm2. Its radius is
(a) 7 cm (b) 5 cm (c) 6 cm (d) 8 cm

2d
86. If radius of a sphere is then its volume is
3
32 3 23 3 32 3 34 3
(a) d (b) d (c) d (d) d
81 4 3 3

87. The capacity of a cylindrical tank is 6160 cm3. Its base diameter is 28 m. The
depth of this tank is
(a) 5 m (b) 10 m (c) 15 m (d) 8 m

88. Base radius of two cylinder are in the ratio 2 : 3 and their heights are in the ratio 5
: 3. The ratio of their volumes is
(a) 27 : 20 (b) 25 : 24 (c) 20 : 27 (d) 15 : 20
89. If base radius and height of a cylinder are increased by 100% then its volume
increased by:
(a) 30% (b) 40% (c) 42% (d) 33.1%
90. The diameter of a sphere is 14 m. The volume of this sphere is
1 1 2 2
(a) 1437 m3 (b) 1357 m3 (c) 1437 m3 (d) 1337 m3
3 3 3 3
Prepared by: M. S. KumarSwamy, TGT(Maths) Page - 65 -
91. The volume of a sphere is 524 cm3. The diameter of sphere is
(a) 5cm (b) 4cm (c) 3cm (d) 7cm
92. The total surface area of a cylinder is 40 cm2. If height is 5.5 cm then its base
radius is
(a) 5cm (b) 2.5cm (c) 1.5cm (d) 10cm
93. The area of circular base of a right circular cone is 78.5 cm2. If its height is 12 cm
then its volume is
(a) 31.4 cm3 (b) 3.14 cm3 (c) 314 cm3 (d) none of these
94. The base radius of a cone is 11.3 cm and curved surface area is 355 cm2. Its height
355
is (Take   )
113
(a) 5 cm (b) 10 cm (c) 11 cm (d) 9 cm

95. The volume and the surface area of a sphere are numerically equal, then the radius
of sphere is
(A) 0 units (B) 1 units (C) 2 units (D) 3 units

96. A cylinder, a cone and a hemisphere are of the same base and of the same height.
The ratio of their volumes is
(A) 1 : 2 : 3 (B) 2 : 1 : 3 (C) 3 : 1 : 2 (D) 3 : 2 : 1

97. Small spheres, each of radius 2cm, are made by melting a solid iron ball of radius
6cm, then the total number of small spheres is
(A) 9 (B) 6 (C) 27 (D) 81

98. Three solid spheres of diameters 6cm, 8cm and 10cm are melted to form a single
solid sphere. The diameter of the new sphere is
(A) 6 cm (B) 4.5 cm (C) 3 cm (D) 12 cm

99. The radii of the ends of a frustum of a cone 40 cm high are 38 cm and 8 cm. The
slant height of the frustum of cone is
(A) 50 cm (B) 10 7 cm (C) 60.96 cm (D) 4 2 cm

100. The circular ends of a bucket are of radii 35 cm and 14 cm and the height of
the bucket is 40 cm. Its volume is
(A) 60060 cm3 (B) 80080 cm3 (C) 70040 cm3 (D) 80160 cm3

101. If the radii of the ends of a bucket are 5 cm and 15 cm and it is 24 cm high,
then its surface area is
(A) 1815.3 cm2 (B) 1711.3 cm2 (C) 2025.3 cm2 (D) 2360 cm2

102. If the radii of the ends of a 42 cm high bucket are 16 cm and 11 cm, determine
22
its capacity (take π  )
7
(A) 24222 cm3 (B) 24332 cm3 (C) 24322 cm3 (D) none of these

Prepared by: M. S. KumarSwamy, TGT(Maths) Page - 66 -


PROBABILITY

IMPORTANT FACTS AND FORMULA

1. Experiment : An operation which can produce some well-defined outcome is


called an experiment

2.Random Experiment: An experiment in which all possible outcome are known and
the exact output cannot be predicted in advance is called an random experiment
Eg of performing random experiment:
(i) Rolling an unbiased dice
(ii)Tossing a fair coin
(iii)Drawing a card from a pack of well shuffled card
(iv)Picking up a ball of certain color from a bag containing ball of different colors

Details:
(i)When we throw a coin. Then either a head(h) or a tail (t) appears.
(ii)A dice is a solid cube, having 6 faces ,marked 1,2,3,4,5,6 respectively when we
throw a die , the outcome is the number that appear on its top face .
(iii)A pack of cards has 52 cards it has 13 cards of each suit ,namely spades, clubs
,hearts and diamonds
Cards of spades and clubs are black cards
Cards of hearts and diamonds are red cards
There are 4 honors of each suit
These are aces ,king ,queen and jack
King, Queen and Jack are called face cards

3.Sample space : When we perform an experiment ,then the set s of all possible
outcome is called the sample space
Eg of sample space:
(i)In tossing a coin ,S={H,T}
(ii)If two coin are tossed ,then S={HH ,TT, HT, TH}.
(iii)In rolling a die we have,s={1,2,3,4,5,6}.
4.Event:any subset of a sample space.
5. Probability of occurrence of an event.
Let S be the sample space and E be the event .
Then, P(E)=n(E)/n(S).
6. Results on probability:
(i) P(sure event) = 1 (ii) 0< P(E)<1 (iii) P(impossible event)=0
(iv) Ffor any event a and b, we have:
P(AB)=P(A)+P(B)-P(AB)
(v)If A denotes (not-A),then P(A)=1 – P( A ).
MCQ QUESTIONS FOR PRACTICE

1. There are 6 marbles in a box with number 1 to6 marked on each of them . What is
the probability of drawing a marble with number 2 ?
1 1 1
(a) (b) (c) (d) 1
6 5 3
Prepared by: M. S. KumarSwamy, TGT(Maths) Page - 67 -
2. A die is thrown once . What will be the probability of getting a prime number ?
1 1
(a) (b) (c) 1 (d) 0
6 2

Cards are marked with numbers 1 to 25 are placed in the box and mixed
thoroughly. One card is drawn at random from the box. Answer the following
questions (Q3-Q12)

3. What is the probability of getting a number 5?


1 1
(a) 1 (b) 0 (c) (d)
25 5

4. What is the probability of getting a number less than 11?


1 2
(a) 1 (b) 0 (c) (d)
5 5

5. What is the probability of getting a number greater than 25?


1 2
(a) 1 (b) 0 (c) (d)
5 5

6. What is the probability of getting a multiple of 5?


1 1
(a) 1 (b) 0 (c) (d)
25 5

7. What is the probability of getting an even number?


12 13
(a) 1 (b) 0 (c) (d)
25 25
8. What is the probability of getting an odd number?
12 13
(a) 1 (b) 0 (c) (d)
25 25
9. What is the probability of getting a prime number?
8 9 12 13
(a) (b) (c) (d)
25 25 25 25

10. What is the probability of getting a number divisible by 3?


8 9 12 13
(a) (b) (c) (d)
25 25 25 25

11. What is the probability of getting a number divisible by 4?


8 9 6 3
(a) (b) (c) (d)
25 25 25 25

12. What is the probability of getting a number divisible by 7?


8 9 6 3
(a) (b) (c) (d)
25 25 25 25

13. A bag has 4 red balls and 2 yellow balls. A ball is drawn from the bag without
looking into the bag. What is probability of getting a red ball?

Prepared by: M. S. KumarSwamy, TGT(Maths) Page - 68 -


1 2 1
(a) (b) (c) (d) 1
6 3 3

14. A bag has 4 red balls and 2 yellow balls. A ball is drawn from the bag without
looking into the bag. What is probability of getting a yellow ball?
1 2 1
(a) (b) (c) (d) 1
6 3 3

A box contains 3 blue, 2 white, and 5 red marbles. If a marble is drawn at


random from the box, then answer the questions from 15 to 19.

15. What is the probability that the marble will be white?


1 1 1
(a) (b) (c) (d) 1
6 5 3

16. What is the probability that the marble will be red?


1 1
(a) (b) (c) 1 (d) 0
6 2

17. What is the probability that the marble will be blue?


3 1
(a) (b) (c) 1 (d) 0
10 2

18. What is the probability that the marble will be any one colour?
1 1
(a) (b) (c) 1 (d) 0
6 2

19. What is the probability that the marble will be red or blue?
4 1 2
(a) 1 (b) (c) (d)
5 5 5

A die is thrown once, then answer the questions from 20 to 24.

20. Find the probability of getting a prime number


1 1
(a) (b) (c) 1 (d) 0
6 2

21. Find the probability of getting a number lying between 2 and 6


1 1
(a) (b) (c) 1 (d) 0
6 2

22. Find the probability of getting an odd number.

1 1
(a) (b) (c) 1 (d) 0
6 2
23. Find the probability of getting an even number.
1 1
(a) (b) (c) 1 (d) 0
6 2

Prepared by: M. S. KumarSwamy, TGT(Maths) Page - 69 -


24. Find the probability of getting a number greater than 4.
1 2 1
(a) (b) (c) (d) 1
6 3 3

A box contains 5 red marbles, 6 white marbles and 4 green marbles. If a marble
is drawn at random from the box, then answer the questions from 25 to 30.

25. What is the probability that the marble will be white?


1 2 1
(a) (b) (c) (d) 1
6 3 3

26. What is the probability that the marble will be red?


1 2 1
(a) (b) (c) (d) 1
6 3 3

27. What is the probability that the marble will be green?


1
(a) 0.3 (b) (c) 1 (d) none of these
2

28. What is the probability that the marble will be any one colour?
1 1
(a) (b) (c) 1 (d) 0
6 2

29. What is the probability that the marble will be red or green?
2 3 1
(a) (b) (c) (d) none of these
5 25 5

30. What is the probability that the marble will be blue?


1 1
(a) (b) (c) 1 (d) 0
6 2

Cards are marked with numbers 1 to 50 are placed in the box and mixed
thoroughly. One card is drawn at random from the box. Answer the following
questions from 31 to 40.

31. What is the probability of getting a number 5?


1 1
(a) 1 (b) 0 (c) (d)
25 5

32. What is the probability of getting a number less than 11?


1 2
(a) 1 (b) 0 (c) (d)
5 5

33. What is the probability of getting a number greater than 50?


1 2
(a) 1 (b) 0 (c) (d)
5 5

34. What is the probability of getting a multiple of 5?


1 1
(a) 1 (b) 0 (c) (d)
25 5
Prepared by: M. S. KumarSwamy, TGT(Maths) Page - 70 -
35. What is the probability of getting an even number?
1 12 13
(a) 1 (b) (c) (d)
2 25 25

36. What is the probability of getting an odd number?


1 12 13
(a) 1 (b) (c) (d)
2 25 25
37. What is the probability of getting a prime number?
1 4 3
(a) 1 (b) (c) (d)
2 10 10

38. What is the probability of getting a number divisible by 3?


8 9 12 13
(a) (b) (c) (d)
25 25 25 25

39. What is the probability of getting a number divisible by 4?


8 9 6 3
(a) (b) (c) (d)
25 25 25 25

40. What is the probability of getting a number divisible by 7?


8 9 6 3
(a) (b) (c) (d)
25 25 25 25

TABULATION AND GRAPH

Tabulation:
Tips for solving tables and graphs problems -
(1)= Read and view tables and diagram properly.
(2)= Put proper attention, what sum rows and sum of columns represents.
(3)= Take care regarding of units.
(4)= Try to understand the question, sometimes you can solve in your mind by just
looking at data.
(5)= Most of the questions can be solved by approximation, thus you can save time
by avoiding calculation.

Bar Graph: This section comprises of questions in which the data collected in a
particular discipline are represented in the form of vertical or horizontal bars drawn
by selecting a particular scale.one of the parameters is plotted on the horizontal axis
and the other on the vertical axis . the candidate is required to understand the given
information and thereafetr answer the given questions on the basis of data analysis.

Pie-Graph: The pie-chart or a pie-graph is a method of representing a given


numerical data in the form of sectors of a circle.
The sectors of the circle are constructed in such a way that the area of each sector is
proportional to the corresponding value of the component of the data.
From geometry, we know that the area of a circle is proportional to the central angle.

Prepared by: M. S. KumarSwamy, TGT(Maths) Page - 71 -


So, the central angle of each sector must be proportional to the corresponding value of
the component.
Since the sum of all the central angle is 360°, we have
Value of the component
Central angle of the component =  3600
Total value
Line graph:This section comprises of question in which the data collected in a
particular discipline are represented by specific points together by straight lines. The
points are plotted on a two-dimensional plane taking one parameter on the horizontal
axis and the other on the vertical axis. The candidate is required to analyse the given
information and thereafter answer the given questions on the basis of the analysis of
data.

Following bar graph shows marks obtained by a student in 2005–06 and 2006–07
subject wise. Read and answer the questions from Q1 – Q10

1. In which subject has the performance improved the most?


(a) Maths (b) Science (c) S. Science (d) none of these

2. In which subject has the performance deteriorated?


(a) English (b) Science (c) S. Science (d) none of these

3. In which subject is the performance at par?


(a) Hindi (b) Science (c) S. Science (d) none of these

4. Find the marks obtained in Maths by a student in 2005–06 ?


(a) 30 (b) 40 (c) 50 (d) 60

5. Find the marks obtained in Maths by a student in 2006–07 ?


(a) 30 (b) 40 (c) 50 (d) 60

6. Find the marks obtained in Hindi by a student in 2005–06 ?


(a) 30 (b) 40 (c) 50 (d) 60

7. Find the marks obtained in Hindi by a student in 2006–07 ?


(a) 30 (b) 40 (c) 50 (d) 60

8. Find the marks obtained in S. Science by a student in 2005–06 ?


Prepared by: M. S. KumarSwamy, TGT(Maths) Page - 72 -
(a) 30 (b) 40 (c) 50 (d) 60

9. Find the total marks obtained by a student in 2005–06?


(a) 230 (b) 235 (c) 240 (d) none of these

10. Find the total marks obtained by a student in 2006–07?


(a) 230 (b) 270 (c) 240 (d) none of these

Frequency Distribution of Daily Income of 550 workers of a factory is given


below. Study the following frequency distribution table and answer the questions
from Q1 – Q10.

Class Interval Frequency


(Daily income in rupees) (Number of workers)
100 – 125 45
125 – 150 25
150 – 175 55
175 – 200 125
200 – 225 140
225 – 250 55
250 – 275 35
275 – 300 50
300 – 325 20
Total 550
1. What is the size of class intervals ?
(a) 24 (b) 25 (c) 26 (d) 15

2. Which class has the highest frequency ?


(a) 200-225 (b) 300-325 (c) 175-200 (d) 150-175

3. Which class has the lowest frequency ?


(a) 100-125 (b) 300-325 (c) 175-200 (d) 150-175

4. What is the upper limit of the class interval 250-275?


(a) 250 (b) 275 (c) 25 (d) 525

5. Which two classes have the same frequency?


(a) III & IV (b) I & II (c) II & V (d) V & VI
6. What is the range of the all class interval?
(a) 250 (b) 275 (c) 225 (d) 525
7. What is the lower limit of the class interval 250-275?
(a) 250 (b) 275 (c) 25 (d) 525
8. What is the total number of workers having daily income less than 250?
(a) 300 (b) 445 (c) 305 (d) 550
9. What is the total number of workers having daily income more than 200?
(a) 300 (b) 445 (c) 305 (d) 550
10. What is the total number of workers having daily income between 150–250?
(a) 300 (b) 445 (c) 375 (d) 550

Prepared by: M. S. KumarSwamy, TGT(Maths) Page - 73 -


The number of hours for which students of particular class watched television
during holidays is shown through the graph given below. See and answer the
questions from Q1 – Q5.

1. For how many hours did the maximum number of students watch TV ?
(a) 4-5 hrs (b) 6-7 hrs (c) 3-4 hrs (d) 2-3hrs

2. How many students watched TV for less than 4 hrs ?


(a) 12 (b) 34 (c) 4 (d) 8

3. How many students spent more than 5 hrs in TV watching ?


(a) 14 (b) 0 (c) 6 (d) 8

4. For how many hours did the minimum number of students watch TV ?
(a) 2-3 hrs (b) 6-7 hrs (c) 1-2 hrs (d) 3-4hrs

5. How many students spent less than 5 hrs in TV watching ?


(a) 34 (b) 32 (c) 8 (d) 66
Adjoining pie-chart gives the expenditure (in %age) on various items and
savings of a family during a month. Study the given pie-chart and answer the
questions from Q6 – Q10.

Prepared by: M. S. KumarSwamy, TGT(Maths) Page - 74 -


6. On which item the expenditure was maximum ?
(a) food (b) education (c) others (d) transport

7. On which item the expenditure was minimum ?


(a) food (b) education (c) others (d) transport
8. Expenditure on which item is equal to total savings of the family ?
(a) food (b) education (c) others (d) transport

9. Expenditure on which item is equal to total savings of the House Rent?


(a) food (b) education (c) clothes (d) transport

If the monthly savings of the family is Rs 3000,


10. What is the monthly income of the family?
(a) 30000 (b) 20000 (c) 25000 (d) 40000

11. What is the monthly expenditure on cloths ?


(a) 3000 (b) 2000 (c) 2500 (d) 1000

12. What is the monthly expenditure on education for children ?


(a) 3000 (b) 2000 (c) 2500 (d) 1000
13. What is the monthly expenditure on education for others?
(a) 3000 (b) 2000 (c) 2500 (d) 4000
14. What is the monthly expenditure on education for Transport?
(a) 3000 (b) 2000 (c) 2500 (d) 1000
15. What is the monthly expenditure on education for Food?
(a) 3000 (b) 5000 (c) 2500 (d) 4000
Ex.1 Production of steel by six different companies in three cosecutive years 1994-95-96( In
Lakh Tonnes ) are being given

Prepared by: M. S. KumarSwamy, TGT(Maths) Page - 75 -


Q(1) What is the difference between average production of the six companies in 1995 and
average production of the same companies in 1994 ?
Solution : Sum of production in 1995= 55+55+40+70+70+45=335
Sum of production in 1994=50+45+30+50+70+35=280
335  280
So, difference   9.16666667
6
Difference=9.16666 tonnes.
Q(2) What is the % decline in production by company C from 1995 to 1996 ?
Solution :For company c production in 1995= 40 and in 1996=35
40  35
% Decline   100  12.5%
40
Q(3) Which of the following companies recorded the minimum % growth from 1994 to 1995 ?
(a) A (b) B (c) C (d) D (e) F
Solution :On seeing graph it is clear that company E has recorded minimum growth from 1994
to 1995
Q(4) Production of company ' C ' in 1995 and production of company ' F ' in 1994 together is
what % of production of B in 1996 ?
Solution : Production of C in 1995 = 40 , Production of F in 1994 = 35
Sum = 75, production of B in 1996 = 60
75
%   100  125%
60
Q(5) In which of the following pairs of companies the difference between average production
for the three years is maximum ?
(a) E and F (b) D and F (c) E and C (d) A and E (e) None of these
Solution : From the graph we can see that highest production average is for company E and
Lowest production average is for company C, so difference between average production for the
three years is maximum for company E and C, So, option (c) is correct
Ex.2 Total sale of English and Hindi Newspapers in Five different Localities of a city are
given. ( Note : Do not get confused by axis of data, it is shown in cylindrical way, so value is
same as shown in y axis )

Prepared by: M. S. KumarSwamy, TGT(Maths) Page - 76 -


Q(1) What is the difference between the total sale of english Newspapers and total sale of
Hindi newspapers in all the localities together ?
(a) 6000 (b) 6500 (c) 70000 (4)7500 (5) None of these
Solution : Total Sum of all the English News papers = 7500 + 9000 + 9500 + 7000 + 6500 =
39500
Total Sum of all the Hindi News papers = 5500 + 8500 + 4500 + 9500 + 5000 = 33000
Difference = 39500 - 33000 = 6500, so option (b) is correct

Q(2) The sale of English News paper in locality A is approximatley what % of the total sale of
english newspapers in all the localities together ?
(a) 527 (b) 25 (c) 111 (4)236 (5) 19
Solution : Total sum of sale of english newspapers in all the localities together = 39500 as
calculated above
Sale of English News paper in locality A = 7500 ( from fig. above )
7500
Required %   500  19
39500
So option (5) is correct.

Q(3) What is the respective ratio of the sale of Hindi Newspapers in locality A to the sale of
Hindi Newspapers in locality D
(a) 11 : 19 (b) 6 : 5 (c) 5 : 6 (4)19 : 11 (5) None of these
Solution :sale of Hindi Newspapers in locality A=5500
sale of Hindi Newspapers in locality D=9500
5500 11
Required ratio  
9500 19
So, option (a) is correct

Q(4) The sale of English Newspaper in localities B and D together is approximately what % of
the sale English Newspaper in localities A, C and E together ?
(a) 162 (b) 84 (c) 68 (4)121 (5) 147
Solution : Sum of the sale of English Newspaper in localities B and D together = 9000 + 7000
= 16000
Sum of the sale of English Newspaper in localities A, C and E together = 7500 + 9500 + 6500
= 23500
16000
Required %   100  68%
23500
So, option (c) is correct

Q(5) What is the average sale of hindi news papers in all the localities together ?

(a) 6600 (b) 8250 (c) 5500 (4)4715 (5) None of these
Solution : Sum of the sale of Hindi Newspapers in all localities = 33000 as calculated in Q(1)
33000
Average   6600
5
So, option (a) correct

Ex.3 - The number of students studying in different faculties in the years 2001 and 2002 from
state A is as follows

Prepared by: M. S. KumarSwamy, TGT(Maths) Page - 77 -


35000

Ex. In which faculty there was decrease in the number of students from 2001 to 2002 ?
(a) Arts (b)Agriculture (c) Pharmacy (d)None (e) None of these

Solution : For Arts, Number of students in 2001 and 2002


12 11
35000   4200, 40000   4400
100 100
For Agriculture Number of students in 2001 and 2002
7 5
35000   2450, 40000   2000
100 100

For pharmacy % is already more and total number of students are already more in 2002, so
correct option will be for Agriculture, option (b) is true

Ex. What is the ratio between the number of students studying pharmacy in the years 2001 and
2002 respectively ?
(a) 4 : 3 (b) 3 : 2 (c) 2 : 3 (d) 7 : 12 (e) None of these
Prepared by: M. S. KumarSwamy, TGT(Maths) Page - 78 -
Solution : Ratio between the number of students studying pharmacy in the years 2001 and 2002
6
35000 
= 100  7
9 12
40000 
100
Option (d) is true

Ex. What was the approximate percentage increase in the number of students of Engineering
from the year 2001 to 2002 ?
(a) 17 (b) 15 (c) 25 (d) 23 (e) 20
Solution : Number of engineering students in 2001 =
18
35000   6300 in 2001
100
19
Number of engineering students in 2002 = 40000   7600 in 2002
100

Total increase = 7600 - 6300 = 1300


1300
% Increase  100  20.63%  20%
6300

Ex. In the year 2001, the number of students studying Arts and Commerce together is what
percentage of the number of students studying these subjects together in 2002 ?
(a) 76 (b) 85 (c) 82 (d) 79 (e) None of these
Solution : Number of students studying Arts and Commerce together in 2001 =
34
35000   11900 in 2001
100

Number of students studying Arts and Commerce together in 2002 =


35
40000   14000 in 2002
100

11900
Required %   100  85%
14000
So, option (b) is true

Ex. In which of the following faculties the percent increase in the number of students
was minimum from 2001 to 2002 ?
(a) Arts (b) Science (c) Commerce (d) Medicine (e) Engineering

Solution : This question is quit lengthy, First find respective number of students in
the given years according to subjects
Arts Science Commerce Medicine Engineering
2001 4200 8400 7700 3850 6300
2002 4400 8800 9600 4000 7600

4400  4200
For arts :  100  4.761%
4200

Prepared by: M. S. KumarSwamy, TGT(Maths) Page - 79 -


8800  8400
For Science :  100  4.761%
8400
9600  7700
For Commerce :  100  24.67%
7700
4000  3850
For Medicine :  100  3.9%
3850
7600  6300
For Engineering : 100  20.63%
6300

Ex. Study the given graph and table and answer the following questions given below. ( Total
population of all states given in Pie chart is 25 lakhs). In the year 1998 the data of different
states regarding population of states

Sex and literacy wise population ratio

States Sex Literacy


M F Literate Illiterate
UP 5 3 2 7
Bihar 3 1 1 4
AP 2 3 2 1
Kar 3 5 3 2
MH 3 4 5 1
TN 3 3 7 2
Kerala 3 4 9 4

Q(1) Approximately what is the total number of literate people in MH and Kar together ?
(a) 4.5 lakhs (b) 6.5 Lakhs (c) 3 lakh (d) 3.5 lakhs (e) 6 lakhs
Solution : Population of Kar is 15 %
25  15 15
Population of Kar =  lakhs
100 4

Prepared by: M. S. KumarSwamy, TGT(Maths) Page - 80 -


So, Total number of literate people in Kar is :
3 15
Literates of Kar =   2.25lakhs
3+2 4

Population of MH is 11 %
25  11 11
Population of MH =  lakhs
100 4
5 11
Literates of MH =   2.29lakhs
5+1 4

So, number of literate people in MH and Kar together = 2.29 + 2.25 = 4.54 lakhs
So, option (a) is correct

Q(2). Approximately what will be the percentage of total male in UP , MH and kerala of the
total population of the given states ?
(a) 20% (b) 18% (c) 28% (d) 30% (e) 25%

Solution : Population in UP is 25 %
25  25 25
Population of UP =   6.25lakhs
100 4
5 25
Male people in UP =   3.93lakhs
5+3 4

Population in MH = 2.75 lakhs from Q(1)


3 11
Male people in MH =   1.17lakhs
3+4 4

Population in Kerala = 8 %
25  8
Population of Kerala =  2lakhs
100
3
Male people in Kerala =  2  0.85lakhs
3+4

So, Sum of male population of UP, MH and kerala = 3.93 + 1.17 + 0.85 = 5.95 lakhs
25 x
5.96   x  24%  25% of total
100
option (e) correct

Ex. Number of hotels in a state, according to years are given ( Study the given chart carefully
and then answer the questions accordingly )

Q(1) The approximate % increase in hotels from year 1989 to 1994 was
(a) 75 (b) 100 (c) 125 (d) 150 (e) 175
Solution :
838  410
%increase   100  104.3  100%
410
Option (b) is correct

Prepared by: M. S. KumarSwamy, TGT(Maths) Page - 81 -


Q(2) If the number of newly made hotels in 1991 was less by 10 then what is the ratio of the
number of hotels in 1991 and that in 1990 ?
(a) 14 : 11 (b) 3 : 4 (c) 4 : 5 (d) 5 : 4 (e) 1 : 4
Solution : No of hotels in 1991 will be 570 - 10 = 560
560 14
Required %  
410 11
Option (a) is correct

Q(3) If the % increase in the number of hotels from 1993 to 1994 continued up to 1995 then
what is the number of hotels built in 1995 ?
(a) Minimum 75 (b) Minimum 70 (c) Minimum 50 (d) Minimum 139 (e) Minimum 80
Solution : First find % increase in 1994 from 1993
838  710
%increase   100  18%
710
Now 18 % of 838 = 150
So, nearest option is (d) Minimum 139 hotels

Q(4) In which of the given years increase in hotels in comparison to the previous year is the
maximum ?
(a) 1990 (b) 1991 (c) 1992 (d) 1993 (e) 1994
Solution:
440  410
In 1990  100  7.31%
410

570  440
In 1991  100  29.54%
440

710  570
In 1992  100  24.56%
570

710  710
In 1993  100  0%
710

Prepared by: M. S. KumarSwamy, TGT(Maths) Page - 82 -


838  710
In 1994  100  18.05%
710

So option (b) is correct

Q(5) If increase in hotels from 1991 to 1992 is P % and increase in hotels from 1992 to 1994 is
Q % , then which of the following relations between P and Q is true ?
(a) Data is inadequate (b) P < Q (c) P = Q (d) P > Q (e) None of these
Solution : P % = 24.56 % from question (4)
Q % = 18.05 %
So P > Q, option (d) correct

Ex. % profit earned by two companies over the years is given in graph. Also
Income  Expenditure
% Profit  100
Expenditure

Ex. If the expenditure of Company B in 2000 was Rs. 200 crores, what was its income ?
(a) Rs. 240 crores (b) Rs. 220 crores (c) Rs. 160 crores (d) Can not be determined (e) None of
these.
Solution : Let income be Rs x crores so, we can use the above formula as
x  200
20   100
200
40 = x - 200
x= 240 crores

Ex. If the income of company A in 2002 was Rs. 600 crores. What was its expenditure ?
(a) Rs. 360 crores (b) Rs. 480 crores (c) Rs. 375 crores (d) Can not be determined (e) None of
these.
Solution : For company A in 2002 % profit was 60 % , Let expenditure be x crores , so
600  x
60  100  x  375crores
x

Prepared by: M. S. KumarSwamy, TGT(Maths) Page - 83 -


Ex. If the income of a company B in 1998 was Rs. 200 crores, what was its profit in 1999 ?
(a) Rs. 21.5 crores (b) Rs. 153 crores (c) Rs. 46.15 crores (d) Can not be determined (e) None
of these.
Solution : Profit can be calculated only when Income and expenditure of the given year should
be known. So, option (d) is correct
Ex. If the Incomes of the two companies in 1998 were equal, what was the ratio of their
Expenditure ?
(a) 1 : 2 (b) 26 : 27 (c) 100 : 67 (d) Can not be determined (e) None of these.
Solution : Let, the income of both companies be P, expenditure of A is E1 and expenditure of
B is E2 Now we can write,
P  E1 100 P
35   E1 
E1 135
P  E2 100 P
30   E2 
E2 130
E1 130 26
 
E2 135 27
So, option (b) is correct
Ex. What is the % increase in % profit for company B from year 2000 to 2001 ?
(a) 75 (b) 175 (c) 42.86 (d) Can not be determined (e) None of these.
35  20
Solution : %increase   100  75%
20

Ex. Study the table and answers the questions.


Financial Statement of A company Over the years (Rupees in Lakhs )
Profit before
Gross Interest Deprciation Net
interest and
Year Turnover Rs. Rs. profit Rs.
depreciation
2000-01 1360 381.9 300 70 10.66
2001-02 1402 403.92 315.35 71.1 18.45
2002-03 1538.4 520.05 390.8 80.01 49.15
2003-04 2116.34 600.02 440.89 89 66
2005-06 2521 810 500.9 91.92 212.8
2006-07 2758.99 920 600 99 220.8

Q (1) During which year did the 'Net Profit ' exceed Rs. 1 crore for the first time ?
(a) 2006-07 (b)2005-06 (c) 2003-04 (d) 2002-03 (e) None of these
Solution :By looking at the table we find option (b) as correct

Q(2) During which year was the " Gross Turnover " closet to thrice the 'Profit before
Interest and Depreciation' ?
(a) 2006-07 (b)2005-06 (c) 2003-04 (d) 2002-03 (e) 2001-02
Solution : Here we have to find out the ratio of "Gross turnover " to the "Profit before
Interest and Depreciation "
for 2006-07 ratio = 2758.99 / 920 = 3.0 , since we get answer by hit and trial of first
option only then we need not have to find other option.
So, correct option is (a) 2006-07

Prepared by: M. S. KumarSwamy, TGT(Maths) Page - 84 -


Q(3) During which of the given year did the 'Net profit ' form the highest proportion of
the ' Profit before Interest and Depreciation ?
(a) 2005-06 (b) 2003-04 (c) 2002-03 (d) 2001-02 (e) 2000-01
Solution : Here we will find the reverse ths ratio between ( Profit before Interest and
Depreciation / Net Profit ) and try to find lowest ratio also we use approximation for
saving time
For 2005-06, 81 / 21 = 4 approx.
For 2003-04, 60 / 6.6 = 9 approx
For 2002-03, 520 / 50 = 10 approx
For 2001-02, 40 / 1.8 = 22 approx
For 2000-01, 380 / 10 = 38 approx
So, lowest ratio is for 2005-06 it means reverse ratio between (Net Profit / Profit before
Interest and Depreciation ) will be highest, so option (a) is correct

Q(4) Which of the following registered the lowest increase in terms of rupees from the
year 2005-06 to the year 2006-07 ?
(a) Gross turnover (b) Profit before interest and depreciation (c) Depreciation
(d) Interest (e) Net Profit

Solution : We calculation with approximation in mind


(a) 2758 - 2520= 2200 approx.
(b) 92 -81 =11= 110 approx
(c) 99 - 91.92 = 08.09 approx
(d) 600 - 505 = 95 approx
(e) 220 - 212= 08 approx
So correct option is (e) Net profit

Q(5) The gross Turnover for 2002-03 is of what percentage of the 'Gross Turnover ' for
2005-06 ?
(a) 61 (b) 163 (c) 0.611 (d) 39 (e) 0.006

Solution : 2002-03 Gross Turnover 1538.40


For 2005-06 Gross Turnover 2521.00
So option (a) is correct

Ex. Marks otained by different students in different subjects


Subject (Maximum Marks )
Students Hindi English Maths Social std. Science Sanskrit Phy. Edu
100 100 100 100 75 50 75
Anupam 85 95 87 87 65 35 71
Bimal 72 97 55 77 62 41 64
Chaman 64 78 74 63 55 25 53
Devendar 65 62 69 81 70 40 50
Girish 92 82 81 79 49 30 61
Vivek 55 70 65 69 44 28 30

Q(1) How many students have scored the lowest marks in two or more subjects ?
(a) 2 (b) 3 (c) 1 (d) 0 (e)4

Prepared by: M. S. KumarSwamy, TGT(Maths) Page - 85 -


Solution :Here option (a) is correct by looking at table we find Chaman and Vivek has
scored lowest marks in two or more subjects
Vivek in Hindi, Science, Phy. Edu
Chaman in Social std, Sanskrit

Q(2) Who has scored the highest marks in all the subjects together ?
(a) Devendar (b) Chaman (c) Anupam
(d) Girish (e)Bimal
Solution : For this calculate total toatal of each students in options
Devendar = 65 + 62 + 69 + 81 + 70 + 40 + 50 = 437
Chaman = 64 + 78 + 74 + 63 + 55 + 25 + 53 = 412
Anupam = 85 + 95 + 87 + 87 + 65 + 35 + 71 = 525
Girish = 92 + 82 + 81 + 79 + 49 + 30 + 61= 474
Bimal = 72 + 97 + 55 + 77 + 62 + 41 + 64 = 468
So correct option is (3) Anupam

Q (3) What is the percentage of Devendar's marks (upto two digits after decimal) in all
the subjects together ?
(a) 88.63 (b) 77.38 (c) 67.83 (d) 62.83 (e)72.83
Solution : Devendar's total marks = 437

437
Required % = 100  72.83
600

Q (4) Marks obtained by Chaman in Hindi are what percentage of marks ( upto two digits
after decimal ) obtained by Anupam in the same subject ?
(a) 75.92 (b) 78.38 (c) 77.29
(d) 75.29 (e)72.83
Solution: Marks obtained in Hindi by Chaman = 64, by Anupam= 85
64
Required % =  100  75.29
85

Q (5) What are the average marks obtained by all the students together in Science ?
(a) 55.75 (b) 57.50 (c) 60.00
(d) 59.50 (e)58.00
Solution : Average maks obtained in Science =
65+62+55+70+49+44 345
Average =   57.5
6 6
So, option (b) true

Prepared by: M. S. KumarSwamy, TGT(Maths) Page - 86 -


PRACTICE PAPER (QUANTITATIVE APTITUDE)
1. If a pipe A fills a tank in 10 hours and a pipe B fills it in 15 hours, then the tank
will be filled by A and B together in how much time?
A. 10 hrs. B. 9 hrs C. 6 hrs D. 8 hrs

2. Two pipes P & Q can separately fill a cistern in 12 minutes and 18 minutes
respectively. In how many minutes can both the pipes fill the cistern, if opened
together?
A. 7 1/5 min B. 7 min C. 7 ½ min D. 10 min

3. Two piped A & B can fill a tank in 36 hrs and 45 hrs respectively. If both pipes are
opened simultaneously, how much time will be taken to fill the tank?
A. 20 hrs B. 21 hrs C. 22 hrs D. None

4. A pipe can fill a tank in 15hrs. Due to leak in the bottom, it is filled in 20 hrs. If the
tank is full, how much time will the leak take to empty it?
A. 55 hrs B. 60 hrs C. 61 hrs D. None

5. If a pipe A fills a tank in 10 hrs and pipe B empties it in 15 15 hrs, then in how
much time will the tank be filled up if A & B are both opened?
A. 4 hrs B. 14 hrs C. 20 hrs D. 30 hrs

6. Two taps A & B can fill a water reservoir in 12 and 15 hrs respectively. How long
would the two taps take to fill this reservoir if both are opened together?
A. 6 ¼ hrs B. 6 ½ hrs C. 7 1/3 hrs D. None

7. Two taps can fill a tank in 24 minutes and 30 minutes respectively. Both of them
are opened together, but the first tap is turned off after 8 minutes. Now find how
long would the second tap take to fill the tank.
A. 24 ½ min B. 12 min C. 13 2/3 min D. 16 min

8. Two pipes A & B can fill a tank in 24 min and 32 min respectively. If both the
pipes are opened simultaneously, the time after which B should be closed so that
tank is filled in 18 minutes would be?
A. 18 min B. 10 min C. 9 min D. 8 min

9. Two pipes A & B can fill a tank in 24 minutes and 36 minutes respectively. If both
the pipes are opened simultaneously, the time after which A should be closed so
that tank is filled in 18 min would be?
A. 12 min B. 10 min C. 9 min D. None

10. Two pipes A & B separately fill a cistern in 12 and 15 min respectively while a 3rd
pipe C can empty it in 10 minutes. How long will it take to fill the cistern if all
pipes are opened?
A. 10 min B. 20 min C. 30 min D. 22 min

11. A pipes fills a tank in 2 hrs and another files the tank in 3hrs but a 3rd pipe empties
the filled tank in 5 hrs. Then if three pipes are opened, the tank will be filled in:
A. 1 hr B. 1 11/19 hr C. 2 1/9 D. None

Prepared by: M. S. KumarSwamy, TGT(Maths) Page - 87 -


1
12. What is the value of: 1
2
1
1
1
1
4
A. 3/2 B. 5/4 C. 5/6 D. 1

1. 342 729 25
  
36 9 196
A. 135/14 B. 18/5 C. 18/7 D. None

14. 9.75 + 25.88 + x = 41.18


A. 5.55 B. 5.75 C. 6.57 D. 4.23

5
15. 30% of 270 + of 64 = x
8
A. 120 B. 81 C. 121 D. 242

16. 73.85 + 215.345 – 167.2134 = x


A. 456.4084 B. 121.6711 C. 120.8296 D. None

17. x% of 150 + 250 = 280


A. 30 B. 10 C. 40 D. 20

18. 25% of 40 ÷4% of 25 = x


A. 1 B. 10 C. 0 D. 2

19. Simplify: (34 * 37) ÷ 310


A. 3 B. 9 C. 27 D. 81

20. Find the value of: 2.70  2.70  4.30  4.30  8.60  2.70
2.70  4.30
A. 6.8 B. 7.6 C. 7.0 D. 8.5

Prepared by: M. S. KumarSwamy, TGT(Maths) Page - 88 -


PRACTICE PAPER (QUANTITATIVE APTITUDE)
Directions (Q. 1 – 10): what should come in place of question mark (?) in the
following questions?

1. (1331)2 ÷ (14641)3 = (11)? × (121)


(a) 2 (b) 4 (c) 6 (d) 8 (e) 10

2. 637.28 – 781.47 + 257.39 = ?


(a) 113.20 (b) 104.30 (c) 122.40 (d) 133.50 (e) None of these

3. 6% of 350 + 2% of 700 = ?% of 1400


(a) 2 (b) 2.5 (c) 3 (d) 4 (e) None of these

4. 4672 ÷ 40 ÷ 4 = ?
(a) 467.2 (b) 29.6 (c) 29.2 (d) 368.8 (e) None of these

5. 7 × ? = 546 ÷ 4
(a) 24.4 (b) 113.5 (c) 37.9 (d) 19.5 (e) None of these

6. 2.4 × (33 ÷ 3.6) = ?


(a) 11.6 (b) 16.3 (c) 23.9 (d) 28.5 (e) None of these

7. (5 × 5 × 5 × 5 × 5)5 × (5 × 5 × 5 × 5)6 ÷ (5 × 5 × 5)7 = (5 × 5)?


(a) 3 (b) 28 (c) 14 (d) 7 (e) None of these
8. 7.2 × 3.6 × 1.5 = ?
(a) 38.88 (b) 36.46 (c) 39.32 (d) 42.18 (e) None of these
9. 13% of 360 + ? = 106
(a) 82.2 (b) 73.6 (c) 61.4 (d) 59.2 (e) None of these
10.39 ÷ 1.3 – 0.7 = ? + 1.6
(a) 13.7 (b) 23.3 (c) 39.3 (d) 27.7 (e) None of these

11.11.8% of 645 = ?
(a) 76.17 (b) 78.11 (c) 89.33 (d) 72.13 (e) None of these

12.49×158÷2227=?
(a) 959 (b) 353 (c) 151 (d) 1151 (e) None of these
13.2268 – 3376 + 5456 = ? × 80
(a) 112.75 (b) 59.45 (c) 54.35 (d) 68.55 (e) None of these
14.579−623 +829=?
(a) 713 (b) 619 (c) 719 (d) 729 (e) None of these
Prepared by: M. S. KumarSwamy, TGT(Maths) Page - 89 -
15.8686 – 6868 = ? × 25
(a) 72.72 (b) 86.86 (c) 68.68 (d) 64.64 (e) None of these

16.4554÷9 + 4792 ÷ 8 = ?
(a) 1000 (b) 1054 (c) 1086 (d) 1105 (e) None of these

17.35% of 172of 1260 + 32% of 650 = ?


(a) 465.75 (b) 456.25 (c) 466.50 (d) 467.00 (e) None of these

18.2201 ÷ 31 = ? ÷ 9
(a) 629 (b) 639 (c) 649 (d) 659 (e) None of these

19.((4184××2323))+−((8140××1313))= ?
(a) 12 (b) 14 (c) 15 (d) 18 (e) None of these

20.166 × 10.5 = ? × 7
(a) 239 (b) 249 (c) 256 (d) 289 (e) None of these

21.The ratio of two numbers is 7 : 5 and the difference between these two
numbers is 16. What is the average of these two numbers?
(a) 40 (b) 48 (c) 56 (d) Cannot be determined(e) None of these

22.Which is the next number in the following series?


0, 7, 26, 63, 124, ?
(a) 168 (b) 195 (c) 210 (d) 215 (e) None of these

23.In how many different ways can the letters of JAGRAN be arranged?
(a) 180 (b) 120 (c) 720 (d) Cannot be determined (e) None of these

24.Five years ago the ratio of the ages of A and B was 2 : 3 and after 7 years it
will be 4 : 5. What is the present age of ‘A’?
(a) 12 years(b) 15 years(c) 17 years(d) Cannot be determined (e) None of these
25.If an amount of `6, 87, 056 is distributed equally amongst 92 persons, how
much amount would each person get?
(a) 8128 (b) 6648 (c) 5698 (d) 7486 (e) 7468
26.What would be the simple interest obtained on an amount of `8220 at the rate
of 7.5% per annum after six years?
(a) Rs. 3, 699 (b) Rs. 3, 798 (c) Rs. 4, 279 (d) Rs. 4, 799 (e) Rs. 2, 681
27.What would be compound interest obtained on an amount of `12600 at the rate
of 10% per annum after two years?
(a) Rs. 2, 520 (b) Rs. 2, 400 (c) Rs. 2, 600 (d) Rs. 2, 646 (e) Rs. 2, 746

Prepared by: M. S. KumarSwamy, TGT(Maths) Page - 90 -


28.The ratio of the ages of Rajesh to that of Ramesh is 5 : 8. After 10 years the
new ratio of their ages will be 7 : 10. What is the age of Ramesh at present?
(a) 20 years (b) 25 years (c) 30 years (d) 35 years (e) 40 years

29.A rectangular room has length 24 metre and breadth 21 metre. What will be
the total cost if the floor is made at `180 per square metre?
(a) Rs. 82, 520 (b) Rs. 82, 820 (c) Rs. 86, 720 (d) Rs. 90, 620
(e) None of these

30.A shopkeeper purchased 25 items at `36 per item. He spent `100 on


transportation of these items. What should be the selling price of each item if he
wants to make 22.5% as profit?
(a) `49 (b) `45 (c) `47 (d) `51 (e) None of these

31.16 men can complete a work in 35 days. In many days can 28 men complete
the same piece of work?
(a) 18 days (b) 20 days (c) 22 days (d) 16 days (e) None of these

32.The cost of 16 kg of rice is `416. The cost of 14 kg of atta is `294 and the cost
of 8 kg of pulse is `352. What will be the total cost of 13 kg rice, 17 kg of atta and
11 kg of pulse?
(a) Rs. 1, 069 (b) Rs. 1, 169 (c) Rs. 1, 179 (d) Cannot be determined
(e) None of these

33.The population of city is 80, 000. If it increases 11% in the first year and
decreases by 9% in the second year, what will be the population of the city after
two years?
(a) 81, 600 (b) 81, 000 (c) 84, 800 (d) Cannot be determined
(e) None of these

34.On selling an item for `96 a man gains 20%. In order to gain 35%. What will
be the increase in the selling price?
(a) Rs. 10 (b) Rs. 12 (c) Rs. 13 (d) Rs. 15 (e) None of these

35.The cost of 21 chairs and 33 tables is `96, 054. Then what will be the cost of
49 chairs and 77 tables?
(a) Rs. 2, 00, 000 (b) Rs. 2, 12, 456 (c) Rs. 2, 24, 126
(d) Data inappropriate (e) None of these

36.The product of two consecutive even numbers is 4, 623. Which is the smaller
number?
(a) 67 (b) 69 (c) 71 (d) 73 (e) 77

Prepared by: M. S. KumarSwamy, TGT(Maths) Page - 91 -


37.A car travels a distance of 120 km at the rate of 30 km/h. It covers the nest 150
km at the rate of 25 km/h and the last 90 km of its journey at the speed of 18
km/h. what is the average speed of the car?
(a) 20 km/h (b) 25 km/h (c) 30 km/h (d) 24 km/h
(e) 35 km/h

38.The average age of a man and his son is 42 years. The ratio of their ages 12
years back was 4 : 1. What is the present age of the son?
(a) 15 years (b) 16 years (c) 20 years (d) 24 years
(e) None of these

39.The sum of money that will give Rs.6 per day (1 year = 365 days) as simple
interest at the rate of 12% per annum is
(a) Rs. 18, 250 (b) Rs. 20, 360 (c) Rs. 22, 720 (d) Rs. 25, 650
(e) Rs. 27, 900

40.A person spends Rs.2, 325 on his monthly expenditure and the rest 85% of his
monthly salary in invested in shares. What is his annual income from salary?
(a) Rs. 1, 76, 000 (b) Rs. 1, 80, 000 (c) Rs. 1, 86, 000
(d) Rs. 1, 96, 000 (e) Rs. 2, 40, 000

Prepared by: M. S. KumarSwamy, TGT(Maths) Page - 92 -

Vous aimerez peut-être aussi